Download as pdf or txt
Download as pdf or txt
You are on page 1of 56

Exam Title : 2019 - Test 15- Environ/S&T...

Email : rahulsreedhar8787@gmail.com
Contact :

Note: If the exam is multi-lingual i.e. English and Hindi. Hindi solutions will be
after the completion of English solutions.
QUESTION 1. MTczODc4K1JhaHVsIFNyZWVkaGFyK3JhaHVsc3JlZWRoYXI4Nzg3QGdtYWlsLmNvbStRV
UVTVElPTiAw
The Intergovernmental Panel on Climate Change (IPCC) is the international body for assessing
the science related to climate change. It was set up by which of the following organisations?

a) WMO and UNEP


b) WMO and UNDP
c) UNESCO
d) WMO and IUCN
Correct Answer: A

Explanation

Solution (a)

The Intergovernmental Panel on Climate Change (IPCC) is the international body for assessing
the science related to climate change. The IPCC was set up in 1988 by the World
Meteorological Organization (WMO) and United Nations Environment Programme
(UNEP ) to provide policymakers with regular assessments of the scientific basis of climate
change, its impacts and future risks, and options for adaptation and mitigation.

IPCC assessments provide a scientific basis for governments at all levels to develop climate
related policies, and they underlie negotiations at the UN Climate Conference the United
Nations Framework Convention on Climate Change (UNFCCC). The assessments are policy-
relevant but not policy-prescriptive: they may present projections of future climate change
based on different scenarios and the risks that climate change poses and discuss the
implications of response options, but they do not tell policymakers what actions to take.

QUESTION 2. MTczODc4K1JhaHVsIFNyZWVkaGFyK3JhaHVsc3JlZWRoYXI4Nzg3QGdtYWlsLmNvbStRV
UVTVElPTiAx
Consider the following statements

1. The United Nations Climate Change Conferences are yearly conferences held in the
framework of the United Nations Framework Convention on Climate Change (UNFCCC).

2. COP 24 is going to be held in Katowice, Poland

Choose the correct code

a) 1 only
b) 2 only
c) Both 1 and 2
d) None of the above
Correct Answer: C
Explanation

IASbaba
Web: http://ilp.iasbaba.com/ Score:
Email: ilp@iasbaba.com 0.00 / 150
Page 1
Exam Title : 2019 - Test 15- Environ/S&T...
Email : rahulsreedhar8787@gmail.com
Contact :

Solution (c)

The United Nations Climate Change Conferences are yearly conferences held in the framework
of the United Nations Framework Convention on Climate Change (UNFCCC). They serve as the
formal meeting of the UNFCCC Parties (Conference of the Parties, COP) to assess progress in
dealing with climate change, and beginning in the mid-1990s, to negotiate the Kyoto Protocol to
establish legally binding obligations for developed countries to reduce their greenhouse gas
emissions.

COP24 is the informal name for the 24th Conference of the Parties to the United Nations
Framework Convention on Climate Change which is going to be held in Katowice, Poland in
December 2018.

The most important task during COP24 will be to develop and adopt a package of decisions
ensuring full implementation of the Paris Agreement, according to the decisions taken in Paris
(COP21), Marrakesh (CMA1.1) and Bonn (COP23). During the summit there will also take place
Supporting Dialogue (Talanoa Dialogue) an overview of the achievements of individual countries
in the field of climate protection.

The Polish presidency plans to focus its message on three key themes:

Technology - development of climate-friendly modern solutions, such as electro mobility;

Man - solidary and just transition of industrial regions;

Nature - achieving climate neutrality by absorbing CO2 by forests and land, or by water
management.

QUESTION 3. MTczODc4K1JhaHVsIFNyZWVkaGFyK3JhaHVsc3JlZWRoYXI4Nzg3QGdtYWlsLmNvbStRV
UVTVElPTiAy
The greenhouse gases warm Earth’s surface by increasing the net downward longwave
radiation reaching the surface. Which amongst the following are greenhouse gases?

1. Chlorofluorocarbons

2. Nitrous oxides

3. Methane

4. Carbon monoxide

5. Water vapour

Choose the correct code

a) 1,2,3 and 4 only


b) 1,2,3 and 5 only
c) 1,2 and 3 only
d) All of the above
Correct Answer: B
Explanation

IASbaba
Web: http://ilp.iasbaba.com/ Score:
Email: ilp@iasbaba.com 0.00 / 150
Page 2
Exam Title : 2019 - Test 15- Environ/S&T...
Email : rahulsreedhar8787@gmail.com
Contact :

Solution (b)

Greenhouse gases are those that absorb and emit infrared radiation in the wavelength range
emitted by Earth. Increasing greenhouse gas emissions cause the greenhouse effect. In order,
the most abundant greenhouse gases in Earth's atmosphere are:

1) Water vapour (H2O)

2) Carbon dioxide (CO2)

3) Methane (CH4)

4) Nitrous oxide (N2O)

5) Ozone (O3)

6) Chlorofluorocarbons (CFCs)

7) Hydrofluorocarbons (incl. HCFCs and HFCs)

CO is not considered a greenhouse gas as it does not absorb the terrestrial radiations.

QUESTION 4. MTczODc4K1JhaHVsIFNyZWVkaGFyK3JhaHVsc3JlZWRoYXI4Nzg3QGdtYWlsLmNvbStRV
UVTVElPTiAz
The difference between insolation (sunlight) absorbed by the Earth and energy radiated back to
space is known as climate forcing. Which of the following mechanisms upset this balance?

1. Fluctuations in earth’s orbit

2. Variation in ocean circulation

3. Change in the composition of earth’s atmosphere

Choose the correct code

a) 1 and 2 only
b) 2 and 3 only
c) 1 and 3 only
d) All of the above
Correct Answer: D

Explanation

Solution (d)

The Earth's climate changes when the amount of energy stored by the climate system is varied.
The most significant changes occur when the global energy balance between incoming energy
from the Sun and outgoing heat from the Earth is upset. There are a number of natural
mechanisms that can upset this balance, for example fluctuations in the Earth's orbit,
variations in ocean circulation and changes in the composition of the Earth's
atmosphere .

IASbaba
Web: http://ilp.iasbaba.com/ Score:
Email: ilp@iasbaba.com 0.00 / 150
Page 3
Exam Title : 2019 - Test 15- Environ/S&T...
Email : rahulsreedhar8787@gmail.com
Contact :

In recent times, the latter has been evident as a consequence not of natural processes but of
man-made pollution, through emissions of greenhouse gases. By altering the global energy
balance, such mechanisms "force" the climate to change. Consequently, scientists call them
"climate forcing" mechanisms.

QUESTION 5. MTczODc4K1JhaHVsIFNyZWVkaGFyK3JhaHVsc3JlZWRoYXI4Nzg3QGdtYWlsLmNvbStRV
UVTVElPTiA0
Consider the following statements

1. Ocean acidification is associated with the increase in the pH of sea water which affects all
the life in the ocean

2. Photosynthetic algae and seagrasses may benefit from ocean acidification, as they require CO
2 to live

Choose the correct code

a) 1 only
b) 2 only
c) Both 1 and 2
d) None of the above
Correct Answer: B

Explanation

Solution (b)

When carbon dioxide (CO 2 ) is absorbed by seawater, chemical reactions occur that reduce
seawater pH, carbonate ion concentration, and saturation states of biologically
important calcium carbonate minerals. These chemical reactions are termed "ocean
acidification" or "OA" for short. Calcium carbonate minerals are the building blocks for the
skeletons and shells of many marine organisms. In areas where most life now congregates in
the ocean, the seawater is supersaturated with respect to calcium carbonate minerals. This
means there are abundant building blocks for calcifying organisms to build their skeletons and
shells. However, continued ocean acidification is causing many parts of the ocean to become
under saturated with these minerals, which is likely to affect the ability of some organisms to
produce and maintain their shells.

Ocean acidification is expected to impact ocean species to varying degrees. Photosynthetic


algae and seagrasses may benefit from higher CO2 conditions in the ocean, as they
require CO2 to live just like plants on land. On the other hand, studies have shown that
lower environmental calcium carbonate saturation states can have a dramatic effect on some
calcifying species, including oysters, clams, sea urchins, shallow water corals, deep sea corals,
and calcareous plankton.

QUESTION 6. MTczODc4K1JhaHVsIFNyZWVkaGFyK3JhaHVsc3JlZWRoYXI4Nzg3QGdtYWlsLmNvbStRV
UVTVElPTiA1

IASbaba
Web: http://ilp.iasbaba.com/ Score:
Email: ilp@iasbaba.com 0.00 / 150
Page 4
Exam Title : 2019 - Test 15- Environ/S&T...
Email : rahulsreedhar8787@gmail.com
Contact :

consider the following statements

1. The ozone layer is a concentration of ozone molecules in the stratosphere.

2. The ozone hole is a rupture in the ozone layer where ozone gas is totally absent.

3. About 90 percent of the planet's ozone is in the ozone layer.

Choose the correct code

a) 1 and 2 only
b) 2 and 3 only
c) 1 and 3 only
d) All of the above
Correct Answer: C

Explanation

Solution (c)

The ozone layer is a concentration of ozone molecules in the stratosphere. About 90 percent of
the planet's ozone is in the ozone layer. Stratospheric ozone is a naturally occurring gas that
filters the sun's ultraviolet radiation. A diminished ozone layer allows more UV radiation to
reach the Earth's surface.

One example of ozone depletion is the annual ozone "hole" over Antarctica that has occurred
during the Antarctic spring since the early 1980s. This is not really a hole through the ozone
layer, but rather a large area of the stratosphere with extremely low amounts of ozone.

Note

It is important to understand that ozone depletion is not limited to the area over the South Pole.
Research has shown that ozone depletion occurs over the latitudes that include North America,
Europe, Asia, and much of Africa, Australia, and South America.

QUESTION 7. MTczODc4K1JhaHVsIFNyZWVkaGFyK3JhaHVsc3JlZWRoYXI4Nzg3QGdtYWlsLmNvbStRV
UVTVElPTiA2
Which of the following are ozone depleting substances?

1. Chlorofluorocarbons

2. Hydro chlorofluorocarbons

3. methyl bromide

4. halons

5. methyl chloroform

Choose the correct code

a) 1,3,4 and 5 only

IASbaba
Web: http://ilp.iasbaba.com/ Score:
Email: ilp@iasbaba.com 0.00 / 150
Page 5
Exam Title : 2019 - Test 15- Environ/S&T...
Email : rahulsreedhar8787@gmail.com
Contact :

b) 1,2,3 and 5 only


c) 1,2,3 and 4 only
d) All of the above
Correct Answer: D

Explanation

Solution (d)

Ozone molecules in the stratosphere are constantly being produced and destroyed by different
types of UV radiation from the sun. Normally, the production and destruction is balanced, so the
amount of ozone in the stratosphere at any given time is stable. However, scientists have
discovered that certain chemicals react with UV radiation in the stratosphere, which causes
them to break apart and release chlorine or bromine atoms. These atoms, in turn, destroy ozone
molecules.

Ozone-depleting substances (ODS), which include chlorofluorocarbons (CFCs) and hydro


chlorofluorocarbons (HCFCs), were once used widely in refrigerants, insulating foams,
solvents, and other applications. These substances all release chlorine atoms into the
stratosphere. A single chlorine atom can break apart more than 100,000 ozone molecules.

Other chemicals that damage the ozone layer include methyl bromide (used as a pesticide),
halons (used in fire extinguishers), and methyl chloroform Methyl chloroform is used as an
industrial solvent. Its ozone depletion potential is 0.11. (used as a solvent in industrial
processes). As methyl bromide and halons are broken apart, they release bromine atoms, which
are 60 times more destructive to ozone molecules than chlorine atoms.

QUESTION 8. MTczODc4K1JhaHVsIFNyZWVkaGFyK3JhaHVsc3JlZWRoYXI4Nzg3QGdtYWlsLmNvbStRV
UVTVElPTiA3
Which of the following is concerned with the Control of Transboundary Movements of
Hazardous Wastes and their Disposal?

a) Minamata convention
b) Basel convention
c) Stockholm convention
d) Vienna convention
Correct Answer: B

Explanation

Solution (b)

The Basel Convention is for the Control of Transboundary Movements of Hazardous Wastes and
Their Disposal. It was designed to reduce the movements of hazardous waste between nations,
and specifically to prevent transfer of hazardous waste from developed to less developed
countries (LDCs). It does not, however, address the movement of radioactive waste.

IASbaba
Web: http://ilp.iasbaba.com/ Score:
Email: ilp@iasbaba.com 0.00 / 150
Page 6
Exam Title : 2019 - Test 15- Environ/S&T...
Email : rahulsreedhar8787@gmail.com
Contact :

QUESTION 9. MTczODc4K1JhaHVsIFNyZWVkaGFyK3JhaHVsc3JlZWRoYXI4Nzg3QGdtYWlsLmNvbStRV
UVTVElPTiA4
Which of the following can be found as pollutants in the drinking water in some parts of India?

1. Arsenic

2. Sorbitol

3. Fluoride

4. Formaldehyde

5. Uranium

Select the correct answer using the codes given below

a) 1 and 3 only
b) 2, 4 and 5 only
c) 1, 3 and 5 only
d) 1, 2, 3, 4 and 5
Correct Answer: C

Explanation

Solution (c)

The source for most drinking water is either rivers or underground aquifers (wells). Since water
can dissolve just about anything that it comes into contact with long enough, often the
groundwater we get isnt pure.

It could contain naturally occurring lead, arsenic, mercury, radium, chloride, iron and copper
compounds dissolved in it.

Iron, which is seldom found in concentrations greater than 10 milligrams per litre (mg/L) or 10
parts per million can be a troublesome chemical in drinking water. Corrosion of pipes is a
common reason why iron is found in drinking water.

Arsenic is a semi-metal found in various foods and mostly in groundwater.

Chlorine and fluoride are added to water to kill pathogens, which are disease producing agents.
An excess amount of chlorine in the water causes a problem because it leaves behind a residue.
This residual amount, when consumed, reacts inside the stomach and damages some cells of
the organs.

Fluoride is added in water just to prevent cavities.

Nitrate, a naturally occurring form of nitrogen, is found in the soil. It is required in large
quantities to sustain high crop yields.

Note: There have been recent evidences of Uranium poisoning in several parts of India.

QUESTION 10. MTczODc4K1JhaHVsIFNyZWVkaGFyK3JhaHVsc3JlZWRoYXI4Nzg3QGdtYWlsLmNvbStRV


UVTVElPTiA5

IASbaba
Web: http://ilp.iasbaba.com/ Score:
Email: ilp@iasbaba.com 0.00 / 150
Page 7
Exam Title : 2019 - Test 15- Environ/S&T...
Email : rahulsreedhar8787@gmail.com
Contact :

Consider the following statements.

1. The Stockholm Convention on Persistent Organic Pollutants is a global treaty whose purpose
is to safeguard human health and the environment from highly harmful chemicals that persist in
the environment and affect the well-being of humans as well as wildlife.

2. The term “dirty dozen” refers to 12 persistent organic pollutants

Choose the correct code

a) 1 only
b) 2 only
c) Both 1 and 2
d) None of the above
Correct Answer: C

Explanation

Solution (c)

The Stockholm Convention on Persistent Organic Pollutants, which was adopted in 2001 and
entered into force in 2004, is a global treaty whose purpose is to safeguard human health and
the environment from highly harmful chemicals that persist in the environment and affect the
well-being of humans as well as wildlife. The Convention requires parties to eliminate and/or
reduce POPs, which have a potential of causing devastating effects such as cancer and
diminished intelligence and have the ability to travel over great distances. The Stockholm
Convention is managed by the United Nations Environment Program and its Secretariat is
based in Geneva, Switzerland.

12 persistent organic pollutants referred to as Dirty dozen were the 12 initial compounds that
were listed under the Stockholm convention.

QUESTION 11. MTczODc4K1JhaHVsIFNyZWVkaGFyK3JhaHVsc3JlZWRoYXI4Nzg3QGdtYWlsLmNvbStRVUVTV


ElPTiAxMA==
Consider the following statements

1. Carbon sequestration is the long-term natural storage of carbon in plants, soils, geologic
formations only

2. The Kyoto Protocol under the United Nations Framework Convention on Climate Change
allows countries to receive credits for their carbon-sequestration activities in the area of land
use, land-use change, and forestry.

Choose the correct code

a) 1 only
b) 2 only
c) Both 1 and 2
d) None of the above
Correct Answer: B

IASbaba
Web: http://ilp.iasbaba.com/ Score:
Email: ilp@iasbaba.com 0.00 / 150
Page 8
Exam Title : 2019 - Test 15- Environ/S&T...
Email : rahulsreedhar8787@gmail.com
Contact :

Explanation

Solution (b)

Carbon sequestration refers to the long-term storage of carbon in plants, soils, geologic
formations, and the ocean. Carbon sequestration occurs both naturally and as a result of
anthropogenic activities and typically refers to the storage of carbon that has the immediate
potential to become carbon dioxide gas.

The Kyoto Protocol under the United Nations Framework Convention on Climate Change allows
countries to receive credits for their carbon-sequestration activities in the area of land use,
land-use change, and forestry as part of their obligations under the protocol. Such activities
could include afforestation (conversion of non-forested land to forest), reforestation (conversion
of previously forested land to forest), improved forestry or agricultural practices, and re-
vegetation.

QUESTION 12. MTczODc4K1JhaHVsIFNyZWVkaGFyK3JhaHVsc3JlZWRoYXI4Nzg3QGdtYWlsLmNvbStRVUVTV


lPTiAxMQ==
Which one of the following is associated with the issue of control and phasing out of the use of
ozone depleting substances?

a) Bretton Woods Conference


b) Montreal Protocol
c) Kyoto Protocol
d) Nagoya Protocol
Correct Answer: B

Explanation

Solution (b)

The Montreal Protocol, finalized in 1987, is a global agreement to protect the stratospheric
ozone layer by phasing out the production and consumption of ozone-depleting substances
(ODS).The Montreal Protocol has proven to be innovative and successful, and is the first treaty
to achieve universal ratification by all countries in the world.

On October 15, 2016, Parties to the Montreal Protocol adopted the Kigali amendment to phase
down production and consumption of hydrofluorocarbons (HFCs) worldwide.

HFCs are widely used alternatives to ozone depleting substances such as


hydrochlorofluorocarbons (HCFCs) and chlorofluorocarbons (CFCs), already controlled under
the Protocol.

QUESTION 13. MTczODc4K1JhaHVsIFNyZWVkaGFyK3JhaHVsc3JlZWRoYXI4Nzg3QGdtYWlsLmNvbStRVUVTV


ElPTiAxMg==
Which of the following are considered to be the best practices to sequester soil carbon?

1. Conservation tillage practices like direct seeding systems

IASbaba
Web: http://ilp.iasbaba.com/ Score:
Email: ilp@iasbaba.com 0.00 / 150
Page 9
Exam Title : 2019 - Test 15- Environ/S&T...
Email : rahulsreedhar8787@gmail.com
Contact :

2. Continuation of summer fallow

3. Extending crop rotations to include perennial forage crops for hay or pasture

Choose the correct code

a) 1 and 2 only
b) 2 and 3 only
c) 1 and 3 only
d) All of the above
Correct Answer: C

Explanation

Solution (c)

Soil conservation practices not only reduce soil erosion but also increase the organic matter
content of soils. Principal conservation strategies, which sequester carbon, include converting
marginal lands to compatible land use systems, restoring degraded soils, and adopting best
management practices. For example, removing agriculturally marginal land from annual crop
production and adopting an ecologically compatible land use, such as livestock grazing and/or
wildlife habitat, can lead to increases in total biomass production and an increase in carbon
content in the soil.

The following best management practices have been proven to sequester soil carbon:

1) Conservation tillage practices like direct seeding systems;

2) Elimination of summer fallow;

3) Extending crop rotations to include perennial forage crops for hay or pasture.

4) management of crop residue and application of organic materials and manures;

5) Soil fertility optimization through improved fertilizer placement and site-specific


management;

6) Other techniques that may improve crop yields, and reduce on-site and off-site production
risks.

QUESTION 14. MTczODc4K1JhaHVsIFNyZWVkaGFyK3JhaHVsc3JlZWRoYXI4Nzg3QGdtYWlsLmNvbStRVUVTV


lPTiAxMw==
Consider the following statements

1. The document "The Future We Want" in which a process to develop a set of SDGs to build
upon the MDGs was adopted in Rio+20 conference on Sustainable development.

2. The United Nations Conference on Sustainable Development - or Rio+20 - took place in Rio
de Janeiro, Brazil

Choose the correct code

IASbaba
Web: http://ilp.iasbaba.com/ Score:
Email: ilp@iasbaba.com 0.00 / 150
Page 10
Exam Title : 2019 - Test 15- Environ/S&T...
Email : rahulsreedhar8787@gmail.com
Contact :

a) 1 only
b) 2 only
c) Both 1 and 2
d) None of the above
Correct Answer: C

Explanation

Solution (c)

The United Nations Conference on Sustainable Development - or Rio+20 - took place in Rio de
Janeiro, Brazil on 20-22 June 2012. It resulted in a focused political outcome document which
contains clear and practical measures for implementing sustainable development.

In Rio, Member States decided to launch a process to develop a set of Sustainable Development
Goals (SDGs), which will build upon the Millennium Development Goals and converge with the
post 2015 development agenda.

The Conference also adopted ground-breaking guidelines on green economy policies.

At the United Nations Conference on Sustainable Development (Rio+20) in Rio de Janeiro,


Brazil, in June 2012, Member States adopted the outcome document "The Future We Want" in
which they decided, inter alia, to launch a process to develop a set of SDGs to build upon the
MDGs and to establish the UN High-level Political Forum on Sustainable Development. The Rio
+20 outcome also contained other measures for implementing sustainable development,
including mandates for future programmes of work in development financing, small island
developing states and more.

QUESTION 15. MTczODc4K1JhaHVsIFNyZWVkaGFyK3JhaHVsc3JlZWRoYXI4Nzg3QGdtYWlsLmNvbStRVUVTV


ElPTiAxNA==
Which of the following are examples of carbon removal geo-engineering schemes?

1. Carbon burial

2. Ocean fertilisation

3. Biochar production

4. Scrubbing towers

Choose the correct code

a) 1,2 and 4 only


b) 2,3 and 4 only
c) 1,2 and 3 only
d) All of the above
Correct Answer: D
Explanation

Solution (d)

IASbaba
Web: http://ilp.iasbaba.com/ Score:
Email: ilp@iasbaba.com 0.00 / 150
Page 11
Exam Title : 2019 - Test 15- Environ/S&T...
Email : rahulsreedhar8787@gmail.com
Contact :

The carbon-removal approach would extract CO2 from other gases in the atmosphere by
changing it into other forms of carbon (such as carbonate) through photosynthesis or artificial
scrubbing. This separated carbon then would be either sequestered in biomass at the surface or
transported away for storage in the ocean or underground. Several carbon-removal
geoengineering schemes have been considered. These include carbon burial, ocean
fertilization, biochar production, and scrubbing towers or artificial trees.

Carbon burial, more commonly known as carbon capture and storage, involves the pumping of
pressurized CO2 into suitable geological structures (that is, with gas-tight upper layers to cap
the buried carbon) deep underground or in the deep ocean.

Ocean fertilization would increase the uptake of CO2 from the air by phytoplankton,
microscopic plants that reside at or near the surface of the ocean.

The production of biochar, a type of charcoal made from animal wastes and plant residues (such
as wood chips, leaves, and husks), can sequester carbon by circumventing the normal
decomposition process or acting as a fertilizer to enhance the sequestration rate of growing
biomass.

Another form of carbon capture would involve the use of scrubbing towers and so-called
artificial trees. In the scrubbing tower method, air would be funnelled into a large, confined
space within the towers by wind-driven turbines. As the air is taken in, it would be sprayed with
one of several chemical compounds, such as sodium hydroxide or calcium hydroxide. These
chemicals would react with the CO2 in the air to form carbonate precipitates and water, and
these by-products could then be piped to safe storage locations.

QUESTION 16. MTczODc4K1JhaHVsIFNyZWVkaGFyK3JhaHVsc3JlZWRoYXI4Nzg3QGdtYWlsLmNvbStRVUVTV


ElPTiAxNQ==
Consider the following statements

1. Bio fertilisers are substances which contains living microorganisms and promotes the growth
by increasing the supply of primary nutrients to the host plant

2. Azatobactor , Clostridium and Rhizobium are examples of nitrogen fixing Bio fertilisers

Choose the correct code

a) 1 only
b) 2 only
c) Both 1 and 2
d) None of the above
Correct Answer: C

Explanation

Solution (c)

Bio fertiliser is a substance which contains living microorganisms which, when applied to seeds,
plant surfaces, or soil, colonize the rhizosphere or the interior of the plant and promotes growth
by increasing the supply or availability of primary nutrients to the host plant. The

IASbaba
Web: http://ilp.iasbaba.com/ Score:
Email: ilp@iasbaba.com 0.00 / 150
Page 12
Exam Title : 2019 - Test 15- Environ/S&T...
Email : rahulsreedhar8787@gmail.com
Contact :

microorganisms in bio fertilizers restore the soil's natural nutrient cycle and build soil organic
matter.

Bio fertilisers are not fertilisers. Fertilisers directly increase the soil fertility by adding
nutrients. Bio fertilizers add nutrients through the natural processes of nitrogen fixation,
solubilizing phosphorus, and stimulating plant growth through the synthesis of growth-
promoting substances. Bio fertilizers can be expected to reduce the use of synthetic fertilizers
and pesticides.

Bio fertilizers provide "eco-friendly" organic agro-input. Nitrogen fixing Bio fertilisers such as
Rhizobium, Clostridium, Azatobactor, Azospirilium and blue green algae (BGA) have been in use
a long time. Rhizobium inoculant is used for leguminous crops. Azatobactor can be used with
crops like wheat, maize, mustard, cotton, potato and other vegetable crops. Azospirillum
inoculations are recommended mainly for sorghum, millets, maize, sugarcane and wheat.

QUESTION 17. MTczODc4K1JhaHVsIFNyZWVkaGFyK3JhaHVsc3JlZWRoYXI4Nzg3QGdtYWlsLmNvbStRVUVTV


ElPTiAxNg==
Consider the following statements with respect to Kyoto protocol

1. Clean Development Mechanism (CDM), encourages developed countries to invest in


technology and infrastructure in less-developed countries, where there are often significant
opportunities to reduce emissions.

2. Emissions trading places an economic value on greenhouse gas emissions

3. Emission limitations are only placed on Annex I countries.

Choose the correct code

a) 1 and 2 only
b) 2 and 3 only
c) 1 and 3 only
d) All of the above
Correct Answer: D

Explanation

Solution (d)

The Kyoto Protocol was adopted as the first addition to the United Nations Framework
Convention on Climate Change (UNFCCC), an international treaty that committed its
signatories to develop national programs to reduce their emissions of greenhouse gases.
Greenhouse gases, such as carbon dioxide (CO2), methane (CH4), nitrous oxide (N2O),
perfluorocarbons (PFCs), hydrofluorocarbons (HFCs), and sulphur hexafluoride (SF6), affect the
energy balance of the global atmosphere in ways expected to lead to an overall increase in
global average temperature, known as global warming.

The Kyoto Protocol separates countries into two groups. Annex I includes developed nations,
while Non-Annex I refers to developing countries. Emission limitations are only placed on
Annex I countries. Non-Annex I nations participate by investing in projects that lower
emissions in their own countries. For these projects, they earn carbon credits. These credits can

IASbaba
Web: http://ilp.iasbaba.com/ Score:
Email: ilp@iasbaba.com 0.00 / 150
Page 13
Exam Title : 2019 - Test 15- Environ/S&T...
Email : rahulsreedhar8787@gmail.com
Contact :

be traded or sold to Annex I countries, which allow them a higher level of maximum carbon
emissions for that period.

The protocol provided several means for countries to reach their targets. One approach was to
make use of natural processes, called sinks, that remove greenhouse gases from the
atmosphere. The planting of trees, which take up carbon dioxide from the air, would be an
example.

Another approach was the international program called the Clean Development Mechanism
(CDM), which encouraged developed countries to invest in technology and
infrastructure in less-developed countries, where there were often significant
opportunities to reduce emissions. Under the CDM, the investing country could claim the
effective reduction in emissions as a credit toward meeting its obligations under the protocol.
An example would be an investment in a clean-burning natural gas power plant to replace a
proposed coal-fired plant.

A third approach was emissions trading, which allowed participating countries to buy and sell
emissions rights and thereby placed an economic value on greenhouse gas emissions .

QUESTION 18. MTczODc4K1JhaHVsIFNyZWVkaGFyK3JhaHVsc3JlZWRoYXI4Nzg3QGdtYWlsLmNvbStRVUVTV


ElPTiAxNw==
Which of the following are the common methods used for solid waste management?

1. Aerobic composting

2. Anaerobic composting

3. Incineration

4. Pyrolysis

5. Sanitary landfill

Choose the correct code

a) 1,3,4 and 5 only


b) 2,3,4 and 5 only
c) 1,2,3 and 5 only
d) All of the above
Correct Answer: D

Explanation

Solution (d)

Waste management is the collection, transportation and disposal of waste materials.

Municipal Solid Waste (Management and Handling) Rules 2000 regulate the management and
handling of the municipal solid wastes and are applicable to every municipal authority
responsible for collection, segregation, storage, transportation, processing and disposal of
municipal solid wastes.

IASbaba
Web: http://ilp.iasbaba.com/ Score:
Email: ilp@iasbaba.com 0.00 / 150
Page 14
Exam Title : 2019 - Test 15- Environ/S&T...
Email : rahulsreedhar8787@gmail.com
Contact :

The commonly practiced technologies for SWM can be grouped under three major categories,
i.e., bio-processing, thermal processing and sanitary landfill. The bio-processing method
includes aerobic and anaerobic composting . Thermal methods are incineration and
pyrolysis. Sanitary landfill is generally used to dispose off the final rejects coming out of the
biological and thermal waste processing units.

QUESTION 19. MTczODc4K1JhaHVsIFNyZWVkaGFyK3JhaHVsc3JlZWRoYXI4Nzg3QGdtYWlsLmNvbStRVUVTV


ElPTiAxOA==
A bio indicator is a living organism that gives us an idea of the health of an ecosystem. Which of
the following indicates air pollution?

a) Lichens
b) Mosses
c) Algae
d) Bryophytes
Correct Answer: A

Explanation

Solution (a)

A bio indicator is a living organism that gives us an idea of the health of an ecosystem. Some
organisms are very sensitive to pollution in their environment, so if pollutants are present, the
organism may change its morphology, physiology or behaviour, or it could even die.

One example of a bio indicator is lichens. These plants, which live on surfaces such as trees or
rocks or soil, are very sensitive to toxins in the air. This is because they obtain their nutrients
mostly from the air. We can tell our forests have clean air by the amount and types of lichens on
the trees. Different species of lichen have different levels of susceptibility to air pollution, so we
can also get an idea of the level of pollution by observing which species are present.

QUESTION 20. MTczODc4K1JhaHVsIFNyZWVkaGFyK3JhaHVsc3JlZWRoYXI4Nzg3QGdtYWlsLmNvbStRVUVTV


ElPTiAxOQ==
Consider the following statements about National Biopharma mission.

1. Innovate in India (i3) is a program under National Biopharma mission to create an enabling
ecosystem to promote entrepreneurship and indigenous manufacturing in the pharmaceuticals
sector.

2. It is a collaboration project with the World Bank.

3. It is implemented by Biotechnology Industry Research Assistance Council (BIRAC), a public


sector undertaking of the Department of Biotechnology.

Choose the correct code

a) 1 and 2 only
b) 2 and 3 only

IASbaba
Web: http://ilp.iasbaba.com/ Score:
Email: ilp@iasbaba.com 0.00 / 150
Page 15
Exam Title : 2019 - Test 15- Environ/S&T...
Email : rahulsreedhar8787@gmail.com
Contact :

c) 1 and 3 only
d) All of the above
Correct Answer: D

Explanation

Solution (d)

The Union Ministry of Science & Technology has launched National Biopharma Mission, a first
ever Industry-Academia mission to accelerate biopharmaceutical development in India.

The program named Innovate in India (I3) is an industry- academia collaborative mission of
Department of Biotechnology (DBT) in collaboration with World Bank for accelerating
discovery research to early development of Biopharmaceuticals and to be implemented
by Biotechnology Industry Research Assistance Council (BIRAC).

The aim of the mission is to enable and nurture an ecosystem for preparing India's
technological and product development capabilities in biopharmaceutical to a level that will be
globally competitive over the next decade, and transform the health standards of India's
population through affordable product development. The program will specifically focus on the
development of new vaccines, bio-therapeutics, diagnostics and medical devices to address the
rising burden of diseases in the country.

QUESTION 21. MTczODc4K1JhaHVsIFNyZWVkaGFyK3JhaHVsc3JlZWRoYXI4Nzg3QGdtYWlsLmNvbStRVUVTV


ElPTiAyMA==
Consider the following statements

1. Brucellosis is an infectious zoonotic disease caused by a type of virus called Brucella.

2. “Brucella free villages” is an initiative of ministry of health to develop novel diagnostics and
vaccines to tackle the disease.

Choose the correct code

a) 1 only
b) 2 only
c) Both 1 and 2
d) None of the above
Correct Answer: D

Explanation

Solution (d)

Brucellosis is an infectious disease caused by a type of bacteria called Brucella. The bacteria
can spread from animals to humans.

India is a brucellosis endemic country with approximately 3 to 5 per cent of cattle and
buffaloes, and around 4 per cent sheep and goats infected with Brucella abortus and Brucella
melitensis respectively. People acquire the infection by consuming unpasteurized milk and other
dairy products, and by coming in contact with the contaminated animal secretions and tissues.

IASbaba
Web: http://ilp.iasbaba.com/ Score:
Email: ilp@iasbaba.com 0.00 / 150
Page 16
Exam Title : 2019 - Test 15- Environ/S&T...
Email : rahulsreedhar8787@gmail.com
Contact :

The Department of Biotechnology (DBT) under Union Ministry of Science & Technology has
launched a pilot project called Brucella Free Villages in 50 villages, Under the project, effort
will be made to develop novel diagnostics and vaccines to tackle the disease.

QUESTION 22. MTczODc4K1JhaHVsIFNyZWVkaGFyK3JhaHVsc3JlZWRoYXI4Nzg3QGdtYWlsLmNvbStRVUVTV


ElPTiAyMQ==
Where is India’s first Coal-gasification fertilizer plant is proposed to come up?

a) Odisha
b) Rajasthan
c) Punjab
d) Madhya Pradesh
Correct Answer: A

Explanation

Solution (a)

Indias first Coal-gasification fertilizer plant is proposed to come up in Talcher, Odisha.

The plant will be built at an estimated investment of Rs. 13,000 crore and is targeted to be
commissioned by 2022. On completion, this plant will have capacity of producing 1.27 Million
Metric Tonnes Per Annum (MMTPA) of Neem coated prilled urea using coal and pet coke as
feedstock. Neem coated urea reduces leaching of nitrogen into soil and checks diversion of urea
from agriculture uses.

QUESTION 23. MTczODc4K1JhaHVsIFNyZWVkaGFyK3JhaHVsc3JlZWRoYXI4Nzg3QGdtYWlsLmNvbStRVUVTV


ElPTiAyMg==
Which of the following toxic compounds are found in e- waste?

1. Lead

2. Neon

3. Cadmium

4. Beryllium

5. Hexavalent chromium

Choose the correct code

a) 1,2,3 and 4 only


b) 1,3,4 and 5 only
c) 2,3,4 and 5 only
d) All of the above
Correct Answer: B

IASbaba
Web: http://ilp.iasbaba.com/ Score:
Email: ilp@iasbaba.com 0.00 / 150
Page 17
Exam Title : 2019 - Test 15- Environ/S&T...
Email : rahulsreedhar8787@gmail.com
Contact :

Explanation

Solution (b)

Electronic waste or e-waste describes discarded electrical or electronic devices. Used


electronics which are destined for reuse, resale, salvage, recycling, or disposal are also
considered e-waste.

Electronic scrap components, such as CPUs, contain potentially harmful materials such as lead,
cadmium, beryllium, Hexavalent chromium or brominated flame retardants.

QUESTION 24. MTczODc4K1JhaHVsIFNyZWVkaGFyK3JhaHVsc3JlZWRoYXI4Nzg3QGdtYWlsLmNvbStRVUVTV


ElPTiAyMw==
Which of the following is correctly matched?

Bio fuels Source

1. First generation bio fuels A) food crops

2. Second generation bio fuels B) Non-food crops

3. Third generation bio fuels C) Algae and cyano bacteria

Choose the correct code

a) 1 only
b) 2 and 3 only
c) 1 and 3 only
d) All of the above
Correct Answer: D

Explanation

Solution (d)

Biofuels are liquid or gaseous fuels produced from biomass that are generally high in sugar
(such as sugarcane, sugar beet, sweet sorghum), starch (such as corn and cassava) or oils (such
as soybeans, rapeseed, coconut, sunflowers, and palms). The two most commonly used biofuels
are ethanol and biodiesel.

First-generation biofuels are made from sugar, starch, vegetable oil, or animal fats using
conventional technology. These are generally produced from grains high in sugar or starch
fermented into bioethanol; or seeds that which are pressed into vegetable oil used in biodiesel.
Common first-generation biofuels include vegetable oils, biodiesel, bio alcohols, biogas, solid
biofuels, syngas.

Second-generation biofuels are produced from non-food crops, such as cellulosic biofuels and
waste biomass (stalks of wheat and corn, and wood). Common second-generation biofuels
include vegetable oils, biodiesel, bio alcohols, biogas, solid biofuels, and syngas. Research
continues on second-generation biofuels including bio hydrogen, bio methanol, DMF, Bio-DME,
Fischer-Tropsch diesel, bio hydrogen diesel, mixed alcohols and wood diesel.

IASbaba
Web: http://ilp.iasbaba.com/ Score:
Email: ilp@iasbaba.com 0.00 / 150
Page 18
Exam Title : 2019 - Test 15- Environ/S&T...
Email : rahulsreedhar8787@gmail.com
Contact :

Third-generation biofuels are produced from extracting oil of algae sometimes referred to as
oilgae. Its production is supposed to be low cost and high-yielding giving up to nearly 30 times
the energy per unit area as can be realized from current, conventional first-generation biofuel
feedstocks.

QUESTION 25. MTczODc4K1JhaHVsIFNyZWVkaGFyK3JhaHVsc3JlZWRoYXI4Nzg3QGdtYWlsLmNvbStRVUVTV


ElPTiAyNA==
Horizon 2020, a joint research programme to develop a "Next Generation Influenza Vaccine" to
protect citizens worldwide is collaboration between India and which of the following countries?

a) US
b) UK
c) China
d) European Union
Correct Answer: D

Explanation

Solution (d)

The Indian government and the European Union (EU), recently joined hands for Rs 240 crore
research programme to develop a "Next Generation Influenza Vaccine " to protect citizens
worldwide. The EU and the Indian government's Department of Biotechnology (DBT), have also
committed 15 million each to fund this joint call for the program named "Horizon 2020".In total,
EUR 30 million (or approximately INR 240 crs.) has been earmarked for research and
innovation actions which aim at advancing the efficacy, safety, duration of immunity, and
reactivity against an increased breadth of influenza strains .

These joint efforts also aim to develop cost-effective and affordable influenza vaccine rapidly
without compromising quality. Keeping this in mind, the participating consortia need to bring
together multi-disciplinary stakeholders who can represent any part of the chain from lab to
market.

QUESTION 26. MTczODc4K1JhaHVsIFNyZWVkaGFyK3JhaHVsc3JlZWRoYXI4Nzg3QGdtYWlsLmNvbStRVUVTV


ElPTiAyNQ==
Moderators are an important component of Nuclear reactors. The function of moderator is to:

a) Absorb the kinetic energy of the neutrons.


b) Absorb the heat generated by the nuclear reaction.
c) Increase the number of neutrons.
d) Increase the speed of neutrons to cause fission.
Correct Answer: A
Explanation

Solution (a)

IASbaba
Web: http://ilp.iasbaba.com/ Score:
Email: ilp@iasbaba.com 0.00 / 150
Page 19
Exam Title : 2019 - Test 15- Environ/S&T...
Email : rahulsreedhar8787@gmail.com
Contact :

A moderator is a material used in a nuclear reactor to slow down the neutrons produced
from fission. By slowing the neutrons down the probability of a neutron interacting with
Uranium-235 nuclei is greatly increased thereby maintaining the chain reaction.

Nuclei with low mass numbers are most effective for this purpose, so the moderator is always a
low-mass-number material. In a fast reactor there is no moderator, only fuel and coolant. The
moderation of neutrons is undesirable in fast reactors. Commonly used moderators include
regular (light) water (roughly 75% of the worlds reactors), solid graphite (20% of reactors)
and heavy water (5% of reactors). Beryllium and beryllium oxide (BeO) have been used
occasionally, but they are very costly.

QUESTION 27. MTczODc4K1JhaHVsIFNyZWVkaGFyK3JhaHVsc3JlZWRoYXI4Nzg3QGdtYWlsLmNvbStRVUVTV


ElPTiAyNg==
Which of the following statements are correct about CERN?

1. CERN is an acronym for European Organization for Nuclear Research.

2. India is a founding member of CERN.

3. The Large Hadron Collider (LHC) in CERN is the World’s largest particle accelerator.

Select the code from below:

a) 1 and 2
b) 2 and 3
c) 1 and 3
d) All of the above
Correct Answer: C

Explanation

Solution (c)

The name CERN is derived from the acronym for the French "Conseil Europen pour la
Recherche Nuclaire", or European Council for Nuclear Research.

In recognition of these substantial contributions, India was granted Observer status to the CER
N Council in 2002. Recently it has been upgraded to Associate member of CERN.

The 27-kilometre LHC is the world's largest particle accelerator. It collides protons or lead ions
at energies approaching the speed of light.

https://home.cern/about

QUESTION 28. MTczODc4K1JhaHVsIFNyZWVkaGFyK3JhaHVsc3JlZWRoYXI4Nzg3QGdtYWlsLmNvbStRVUVTV


ElPTiAyNw==
Which of the following correctly explains the term ‘Nuclear Transmutation’?

a) Use of radioactive materials to cause mutations in humans

IASbaba
Web: http://ilp.iasbaba.com/ Score:
Email: ilp@iasbaba.com 0.00 / 150
Page 20
Exam Title : 2019 - Test 15- Environ/S&T...
Email : rahulsreedhar8787@gmail.com
Contact :

b) Conversion of one chemical element or isotope into another


c) Chemical reaction of two compounds to form a new compound
d) None of the above
Correct Answer: B

Explanation

Solution (b)

Nuclear transmutation is the conversion of one chemical element or an isotope into another.
This can occur naturally through radioactive decay or it can be done through nuclear reactions.

QUESTION 29. MTczODc4K1JhaHVsIFNyZWVkaGFyK3JhaHVsc3JlZWRoYXI4Nzg3QGdtYWlsLmNvbStRVUVTV


ElPTiAyOA==
A large energy is released in a nuclear reaction because,

a) Bonds are broken between the elements as they are hit by neutrons.
b) Kinetic energy of neutrons is converted into heat energy
c) Mass gets converted into energy
d) All of the above
Correct Answer: C

Explanation

Solution (c)

During a nuclear reaction, some mass gets converted into energy.

The energy generated can be calculated with the help of Einsteins famous equation

E = MC 2 , where

E = Energy generated

M = Mass converted

C = Speed of light

QUESTION 30. MTczODc4K1JhaHVsIFNyZWVkaGFyK3JhaHVsc3JlZWRoYXI4Nzg3QGdtYWlsLmNvbStRVUVTV


ElPTiAyOQ==
Which of the following statements is INCORRECT about Nuclear Fusion?

a) A lot of heat is required for the reaction to occur.


b) Nuclear fusion is responsible for heat in stars.
c) It releases a large amount of radioactive waste.
d) Very light nuclei are fused to give a heavy nucleus.
Correct Answer: C

IASbaba
Web: http://ilp.iasbaba.com/ Score:
Email: ilp@iasbaba.com 0.00 / 150
Page 21
Exam Title : 2019 - Test 15- Environ/S&T...
Email : rahulsreedhar8787@gmail.com
Contact :

Explanation

Solution (c)

In case of nuclear fusion, two small nuclei are fused to form a heavier element.

Since radioactive materials are not playing any role, radioactive waste is not generated.

nuclear fusion is a reaction in which two or more atomic nuclei come close enough to form
one or more different atomic nuclei and subatomic particles (neutrons or protons). The
difference in mass between the products and reactants is manifested as the release of large
amounts of energy.

QUESTION 31. MTczODc4K1JhaHVsIFNyZWVkaGFyK3JhaHVsc3JlZWRoYXI4Nzg3QGdtYWlsLmNvbStRVUVTV


ElPTiAzMA==
ITER (International Thermonuclear Experimental Reactor) is the world’s largest nuclear fusion
experiment to harness fusion energy. Which of the following statements are correct about ITER?

1. It is using a magnetic confinement ( Toakamak ) to confine the reaction.

2. Hydrogen is being used as a fuel in ITER.

3. India is one of the member countries participating in ITER.

Select the code from below:

a) 1 only
b) 1 and 3
c) 2 and 3
d) All of the above
Correct Answer: D
Explanation

Solution (d)

IASbaba
Web: http://ilp.iasbaba.com/ Score:
Email: ilp@iasbaba.com 0.00 / 150
Page 22
Exam Title : 2019 - Test 15- Environ/S&T...
Email : rahulsreedhar8787@gmail.com
Contact :

ITER ( International Thermonuclear Experimental Reactor , also Latin for "way") is an


international nuclear fusion research and engineering megaproject, which will be the world's
largest magnetic confinement plasma physics experiment. It is an experimental tokamak
nuclear fusion reactor that is being built next to the Cadarache facility in Saint-Paul-ls-Durance,
which is in southern France.

The ITER fusion reactor has been designed to produce 500 megawatts of output power for
around twenty minutes while needing 50 megawatts to operate.

The project is funded and run by seven member entitiesthe European Union, India, Japan,
China, Russia, South Korea, and the United States. The EU, as host party for the ITER complex,
is contributing about 45 percent of the cost, with the other six parties contributing
approximately 9 percent each.

QUESTION 32. MTczODc4K1JhaHVsIFNyZWVkaGFyK3JhaHVsc3JlZWRoYXI4Nzg3QGdtYWlsLmNvbStRVUVTV


ElPTiAzMQ==
Jaitapur Nuclear Power Project is a proposed 9900 MW power project of Nuclear Power
Corporation of India (NPCIL). Consider the following statements regarding this:

1. If built, it would be the largest nuclear power generating station in the world by net electrical
power rating.

2. It is being built with the help of a Russian firm, Areva .

3. The complete farm will have ten Voda Voda nuclear reactors.

Which of the above statements are correct?

a) 1 only
b) 1 and 2
c) 2 and 3
d) All of the above
Correct Answer: A

Explanation

Solution (a)

Jaitapur Nuclear Power Project is a proposed 9900 MW power project of Nuclear Power
Corporation of India (NPCIL) at Madban village of Ratnagiri district in Maharashtra. If built, it
would be the largest nuclear power generating station in the world by net electrical power
rating.

French state-controlled nuclear engineering firm Areva S.A. and Indian state-owned nuclear
operator Nuclear Power Corporation of India signed the agreement, valued about $9.3 billion.

It is proposed to construct 6 European Pressurized Reactors designed and developed by Areva


of France, each of 1650 MW, thus totaling 9900 MW. These are the third generation pressurised
water reactors (PWR).

IASbaba
Web: http://ilp.iasbaba.com/ Score:
Email: ilp@iasbaba.com 0.00 / 150
Page 23
Exam Title : 2019 - Test 15- Environ/S&T...
Email : rahulsreedhar8787@gmail.com
Contact :

QUESTION 33. MTczODc4K1JhaHVsIFNyZWVkaGFyK3JhaHVsc3JlZWRoYXI4Nzg3QGdtYWlsLmNvbStRVUVTV


ElPTiAzMg==
Consider the following statements:

1. Genetic Engineering Appraisal Committee (GEAC) is top biotech regulator in India which
comes under the Ministry of Environment and Forest.

2. Currently Bt Cotton is the only GM crop which is being cultivated in India.

Which of the above statements are correct?

a) 1 only
b) 2 only
c) Both 1 and 2
d) Neither 1 nor 2
Correct Answer: C

Explanation

Solution (c)

The top biotech regulator in India is Genetic Engineering Appraisal Committee (GEAC). The
committee functions as a statutory body under the Environment Protection Act 1986 of the
Ministry of Environment & Forests (MoEF). It was earlier known as Genetic Engineering
Approval Committee. Under the EPA 1986 Rules for Manufacture, Use, Import, Export and
Storage of Hazardous Microorganisms/Genetically Engineered Organisms or Cells , GEAC is
responsible for granting permits to conduct experimental and large-scale open field trials and
also grant approval for commercial release of biotech crops.

The country has yet to approve commercial cultivation of a GM food crop. The only genetically
modified cash crop under commercial cultivation in India is cotton.

1) Bt Cotton For the time being, the only genetically modified crop that is under cultivation in
India is Bt cotton which is grown over 10.8 million hectares. Bt cotton was first used in India in
2002.

2) Bt Brinjal The GEAC in 2007, recommended the commercial release of Bt Brinjal, which was
developed by Mahyco (Maharashtra Hybrid Seeds Company) in collaboration with the
Dharward University of Agricultural sciences and the Tamil Nadu Agricultural University. But
the initiative was blocked in 2010.

3) GM Mustard GEAC has recently given a go ahead for tests of GM mustard before taking a
decision on commercialization.

QUESTION 34. MTczODc4K1JhaHVsIFNyZWVkaGFyK3JhaHVsc3JlZWRoYXI4Nzg3QGdtYWlsLmNvbStRVUVTV


ElPTiAzMw==
Which of the following statements are correct about DNA and RNA?

1. Both DNA and RNA are double stranded.

IASbaba
Web: http://ilp.iasbaba.com/ Score:
Email: ilp@iasbaba.com 0.00 / 150
Page 24
Exam Title : 2019 - Test 15- Environ/S&T...
Email : rahulsreedhar8787@gmail.com
Contact :

2. All four Bases present in DNA and RNA are same, but their combination is different.

Which of the above statements are correct?

a) 1 only
b) 2 only
c) Both 1 and 2
d) Neither 1 nor 2
Correct Answer: D
Explanation

Solution (d)

DNA RNA

Structural
Deoxyribonucleic Acid Ribonucleic Acid
Name:

Transfer the genetic code needed for


the creation of proteins from the
Medium of long-term storage and nucleus to the ribosome. This process
Function:
transmission of genetic information. prevents the DNA from having to leave
the nucleus, so it stays safe. Without
RNA, proteins could never be made.

A single-stranded molecule in most of


Typically a double- stranded molecule with
Structure: its biological roles and has a shorter
a long chain of nucleotides.
chain of nucleotides.

Long polymer with a deoxyribose and Shorter polymer with a ribose and
Bases/ phosphate backbone and four different phosphate backbone and four different
Sugars: bases: adenine, guanine, cytosine and bases: adenine, guanine, cytosine, and
thymine. uracil.

Base A-T (Adenine-Thymine), G-C (Guanine- A-U (Adenine-Uracil), G-C (Guanine-


Pairing: Cytosine) Cytosine)

Deoxyribose sugar in DNA is less reactive Ribose sugar is more reactive because
because of C-H bonds. Stable in alkaline of C-OH (hydroxyl) bonds. Not stable
conditions. DNA has smaller grooves in alkaline conditions. RNA on the
Stability:
where the damaging enzyme can attach other hand has larger grooves which
which makes it harder for the enzyme to makes it easier to be attacked by
attack DNA. enzymes.

IASbaba
Web: http://ilp.iasbaba.com/ Score:
Email: ilp@iasbaba.com 0.00 / 150
Page 25
Exam Title : 2019 - Test 15- Environ/S&T...
Email : rahulsreedhar8787@gmail.com
Contact :

The helix geometry of DNA is of B-Form. The helix geometry of RNA is of A-


DNA is completely protected by the body Form. RNA strands are continually
Unique
i.e. the body destroys enzymes that cleave made, broken down and reused. RNA
Traits:
DNA. DNA can be damaged by exposure is more resistant to damage by Ultra-
to Ultra-violet rays. violet rays.

QUESTION 35. MTczODc4K1JhaHVsIFNyZWVkaGFyK3JhaHVsc3JlZWRoYXI4Nzg3QGdtYWlsLmNvbStRVUVTV


ElPTiAzNA==
Potency of a cell is its ability to develop into differentiated cells. In the light of this statement,
consider the following statements:

1. Totipotent cells have the ability to create an entire organism.

2. Embryonic stem cells are totipotent.

Which of the above statements are correct?

a) 1 only
b) 2 only
c) Both 1 and 2
d) Neither 1 nor 2
Correct Answer: C

Explanation

Solution (c)

Totipotency is the ability of a single cell to divide and produce all of the differentiated cells in an
organism. Spores and zygotes are examples of totipotent cells. In the spectrum of cell potency, t
otipotency represents the cell with the greatest differentiation potential.

Totipotent cells can form all the cell types in a body, plus the extraembryonic, or placental,
cells. Embryonic cells within the first couple of cell divisions after fertilization are the only cells
that are totipotent. Pluripotent cells can give rise to all of the cell types that make up the body;
embryonic stem cells are considered pluripotent. Multipotent cells can develop into more than
one cell type, but are more limited than pluripotent cells; adult stem cells and cord blood stem
cells are considered multipotent.

QUESTION 36. MTczODc4K1JhaHVsIFNyZWVkaGFyK3JhaHVsc3JlZWRoYXI4Nzg3QGdtYWlsLmNvbStRVUVTV


ElPTiAzNQ==
Cloning is the process of producing similar populations of genetically identical individuals.
Which of the following statements are correct about cloning?

1. Cloning does not occur naturally.

2. Cloning of organisms can occur through asexual reproduction.

3. Dolly, a sheep, was the first mammal to be artificially cloned.

IASbaba
Web: http://ilp.iasbaba.com/ Score:
Email: ilp@iasbaba.com 0.00 / 150
Page 26
Exam Title : 2019 - Test 15- Environ/S&T...
Email : rahulsreedhar8787@gmail.com
Contact :

Select the code from following:

a) 1 and 2
b) 2 and 3
c) 1 and 3
d) All of the above
Correct Answer: B

Explanation

Solution (b)

In biology, cloning is the process of producing similar populations of genetically identical


individuals that occurs in nature when organisms such as bacteria, insects or plants reproduce
asexually. Cloning in biotechnology refers to processes used to create copies of DNA fragments
(molecular cloning), cells (cell cloning), or organisms.

Reproductive cloning generally uses "somatic cell nuclear transfer" (SCNT) to create animals
that are genetically identical. This process entails the transfer of a nucleus from a donor adult
cell (somatic cell) to an egg from which the nucleus has been removed, or to a cell from a
blastocyst from which the nucleus has been removed. If the egg begins to divide normally it is
transferred into the uterus of the surrogate mother. Such clones are not strictly identical since
the somatic cells may contain mutations in their nuclear DNA. Additionally, the mitochondria in
the cytoplasm also contains DNA and during SCNT this mitochondrial DNA is wholly from the
cytoplasmic donor's egg, thus the mitochondrial genome is not the same as that of the nucleus
donor cell from which it was produced.

Dolly was the first mammal to have been successfully cloned from an adult somatic cell.

QUESTION 37. MTczODc4K1JhaHVsIFNyZWVkaGFyK3JhaHVsc3JlZWRoYXI4Nzg3QGdtYWlsLmNvbStRVUVTV


ElPTiAzNg==
Consider the following statements

1. Blue carbon is the carbon captured by the world's oceans and coastal ecosystems.

2. Blue Carbon Initiative is coordinated by Conservation International (CI), the International


Union for Conservation of Nature (IUCN) only

3. Coastal blue carbon ecosystems are found along the coasts of every continent except
Antarctica

Select the correct statements

a) 1 and 2
b) 2 and 3
c) 1 and 3
d) All of the above
Correct Answer: C
Explanation

IASbaba
Web: http://ilp.iasbaba.com/ Score:
Email: ilp@iasbaba.com 0.00 / 150
Page 27
Exam Title : 2019 - Test 15- Environ/S&T...
Email : rahulsreedhar8787@gmail.com
Contact :

Solution (c)

Blue carbon is the carbon captured by the world's oceans and coastal ecosystems. The carbon
captured by living organisms in oceans is stored in the form of biomass and sediments from
mangroves, salt marshes, seagrasses and potentially algae.

Coastal blue carbon ecosystems are found along the coasts of every continent except
Antarctica. Mangroves grow in the intertidal zone of tropical and subtropical shores. Countries
with the highest areas of mangroves include Indonesia, Australia, Mexico, Brazil, Nigeria,
Malaysia, Myanmar, Papua New Guinea, Cuba, India, Bangladesh, and Mozambique.

Tidal marshes are intertidal ecosystems occurring on sheltered coastlines ranging from the sub-
arctic to the tropics, though most extensively in temperate zones, mainly in Europe, North-
America, Australia and in the higher latitudes of South-America and Africa.

Seagrass meadows are communities of underwater-flowering plants found in coastal waters of


all continents except Antarctica. More than 60 seagrass species are known to exist, and as
many as 10 to 13 of them may co-occur in tropical sites.

While several countries are embarking on efforts to better map and quantify these systems,
regional, and global maps of coastal blue carbon hot spots currently do not exist.

QUESTION 38. MTczODc4K1JhaHVsIFNyZWVkaGFyK3JhaHVsc3JlZWRoYXI4Nzg3QGdtYWlsLmNvbStRVUVTV


ElPTiAzNw==

IASbaba
Web: http://ilp.iasbaba.com/ Score:
Email: ilp@iasbaba.com 0.00 / 150
Page 28
Exam Title : 2019 - Test 15- Environ/S&T...
Email : rahulsreedhar8787@gmail.com
Contact :

A new concept of Space Based Solar Power (SBSP) is an idea under consideration to generate
clean energy. Which of the following statements is/are correct regarding SBSP?

1. It is the concept of collecting solar power in outer space and distributing it to Earth.

2. Solar collectors will collect the light in space and beam it back to earth as microwaves.

Select the code from following:

a) 1 only
b) 2 only
c) Both 1 and 2
d) Neither 1 nor 2
Correct Answer: C

Explanation

Solution (c)

Space-based solar power (SBSP)

Space-based solar power (SBSP) is the concept of collecting solar power in outer space and
distributing it to Earth. Potential advantages of collecting solar energy in space include a higher
collection rate and a longer collection period due to the lack of a diffusing atmosphere, and the
possibility of placing a solar collector in an orbiting location where there is no night. A
considerable fraction of incoming solar energy (5560%) is lost on its way through the Earth's
atmosphere by the effects of reflection and absorption.

Space-based solar power systems convert sunlight to microwaves outside the atmosphere,
avoiding these losses and the downtime due to the Earth's rotation, but at great cost due to the
expense of launching material into orbit. SBSP is considered a form of sustainable or green
energy, renewable energy, and is occasionally considered among climate engineering proposals.
It is attractive to those seeking large-scale solutions to anthropogenic climate change or fossil
fuel depletion (such as peak oil).

Think

Sun tower concept

QUESTION 39. MTczODc4K1JhaHVsIFNyZWVkaGFyK3JhaHVsc3JlZWRoYXI4Nzg3QGdtYWlsLmNvbStRVUVTV


ElPTiAzOA==
Consider the following statements regarding hydrogen fuel cells:

1. It is a clean energy device which converts chemical energy into electrical energy.

2. The by-product of hydrogen fuel cell is Carbon dioxide.

Which of the above statements is/are correct?

a) 1 only
b) 2 only

IASbaba
Web: http://ilp.iasbaba.com/ Score:
Email: ilp@iasbaba.com 0.00 / 150
Page 29
Exam Title : 2019 - Test 15- Environ/S&T...
Email : rahulsreedhar8787@gmail.com
Contact :

c) Both 1 and 2
d) Neither 1 nor 2
Correct Answer: A

Explanation

Solution (a)

Hydrogen Fuel cells

A fuel cell combines hydrogen and oxygen to produce electricity, heat, and water. Fuel cells are
often compared to batteries. Both convert the energy produced by a chemical reaction into
usable electric power. However, the fuel cell will produce electricity as long as fuel (hydrogen)
is supplied, never losing its charge.

Hydrogen is high in energy, yet an engine that burns pure hydrogen produces almost no
pollution. NASA has used liquid hydrogen since the 1970s to propel the space shuttle and other
rockets into orbit. Hydrogen fuel cells power the shuttle's electrical systems, producing a clean
by-product - pure water, which the crew drinks.

Fuel cells are a promising technology for use as a source of heat and electricity for buildings,
and as an electrical power source for electric motors propelling vehicles. Fuel cells operate best
on pure hydrogen. But fuels like natural gas, methanol, or even gasoline can be reformed to
produce the hydrogen required for fuel cells.

Think

Extraction of Hydrogen

Reforming

QUESTION 40. MTczODc4K1JhaHVsIFNyZWVkaGFyK3JhaHVsc3JlZWRoYXI4Nzg3QGdtYWlsLmNvbStRVUVTV


ElPTiAzOQ==
In recent times, there is a rise in the exploration works of the shale gas. Which of the following
statements are correct regarding ‘Shale Gas’?

1. It is predominantly Methane trapped in Shale rock formations.

2. The source of formation of shale gas is somewhere else and it travels through the permeable
shale rocks and gets trapped in the pores.

3. Hydraulic Fracturing is the technique used for extraction of Shale gas.

Which of the above statements are correct?

a) 1 and 2
b) 2 and 3
c) 1 and 3
d) All of the above
Correct Answer: C
Explanation

IASbaba
Web: http://ilp.iasbaba.com/ Score:
Email: ilp@iasbaba.com 0.00 / 150
Page 30
Exam Title : 2019 - Test 15- Environ/S&T...
Email : rahulsreedhar8787@gmail.com
Contact :

Solution (c)

Shale Gas

Shale gas refers to natural gas that is trapped within shale formations. Shales are fine-grained
sedimentary rocks that can be rich sources of petroleum and natural gas.

Over the past decade, the combination of horizontal drilling and hydraulic fracturing has
allowed access to large volumes of shale gas that were previously uneconomical to produce.

Hydraulic Fracturing

Hydraulic fracturing (commonly called "fracking" or "hydrofracking") is a technique in which


water, chemicals, and sand are pumped into the well to unlock the hydrocarbons trapped in
shale formations by opening cracks (fractures) in the rock and allowing natural gas to flow from
the shale into the well. When used in conjunction with horizontal drilling, hydraulic fracturing
enables gas producers to extract shale gas at reasonable cost. Without these techniques,
natural gas does not flow to the well rapidly, and commercial quantities cannot be produced
from shale.

Shale Gas vs. Conventional Gas

Conventional gas reservoirs are created when natural gas migrates toward the Earth's surface
from an organic-rich source formation into highly permeable reservoir rock, where it is trapped
by an overlying layer of impermeable rock. In contrast, shale gas resources form within the
organic-rich shale source rock. The low permeability of the shale greatly inhibits the gas from
migrating to more permeable reservoir rocks. Without horizontal drilling and hydraulic
fracturing, shale gas production would not be economically feasible because the natural gas
would not flow from the formation at high enough rates to justify the cost of drilling.

QUESTION 41. MTczODc4K1JhaHVsIFNyZWVkaGFyK3JhaHVsc3JlZWRoYXI4Nzg3QGdtYWlsLmNvbStRVUVTV


lPTiA0MA==
Which of the following statements correctly defines cold fusion?

a) It is fusion of two molten metals at room temperature.


b) It refers to a nuclear fusion reaction taking place at cryogenic temperatures.
c) It refers to nuclear fusion reaction taking place at room temperature.
d) It refers to nuclear fusion reaction in the stars.
Correct Answer: C

Explanation

Solution (c)

Cold Fusion

Cold fusion is a hypothesized type of nuclear reaction that would occur at, or near, room
temperature. This is compared with the "hot" fusion which takes place naturally within stars,
under immense pressure and at temperatures of millions of degrees, and distinguished from
muon-catalyzed fusion. There is currently no accepted theoretical model that would allow cold
fusion to occur.

IASbaba
Web: http://ilp.iasbaba.com/ Score:
Email: ilp@iasbaba.com 0.00 / 150
Page 31
Exam Title : 2019 - Test 15- Environ/S&T...
Email : rahulsreedhar8787@gmail.com
Contact :

QUESTION 42. MTczODc4K1JhaHVsIFNyZWVkaGFyK3JhaHVsc3JlZWRoYXI4Nzg3QGdtYWlsLmNvbStRVUVTV


lPTiA0MQ==
Endosulfan has been used as a pesticide in agriculture for a long time. India has agreed to
phase out completely the use and manufacture of endosulfan by 2017. Which of the following
statements are correct about endosulfan ?

1. It is not pest specific and it can negatively impact the populations of beneficial insects.

2. Endosulfan is acutely neurotoxic to both insects and mammals.

3. Endosulfan has high potential to bio-accumulate and bio-magnify.

4. Endosulfan is banned across the world under Stockholm convention.

5. Endosulfan is the cause of death of Indian vultures.

Select the code from the following:

a) 1,2 and 3
b) 1,3 and 4
c) 2,3,4 and 5
d) All of the above
Correct Answer: A

Explanation

Solution (a)

Endosulfan ban is being discussed under Stockholm convention for Persistent Organic
Pollutants but nothing is yet decided.

Cause of death of Indian vultures is the use of diclofinac.

QUESTION 43. MTczODc4K1JhaHVsIFNyZWVkaGFyK3JhaHVsc3JlZWRoYXI4Nzg3QGdtYWlsLmNvbStRVUVTV


ElPTiA0Mg==
Consider the following statements:

1. Agenda 21 is a binding action plan of the United Nations with regard to Sustainable
Development.

2. It is a product of Earth Summit held at Rio de Jeneiro in 1992.

3. ‘21’ in Agenda 21 refers to 21 points of the document that has been pledged for sustainable
development.

Which of the above statements are incorrect?

a) 1 and 2
b) 2 and 3

IASbaba
Web: http://ilp.iasbaba.com/ Score:
Email: ilp@iasbaba.com 0.00 / 150
Page 32
Exam Title : 2019 - Test 15- Environ/S&T...
Email : rahulsreedhar8787@gmail.com
Contact :

c) 1 and 3
d) None of the above
Correct Answer: C

Explanation

Solution (c)

Agenda 21 is a non-binding, voluntarily implemented action plan of the United Nations with
regard to sustainable development. It is a product of the Earth Summit (UN Conference on
Environment and Development) held in Rio de Janeiro, Brazil, in 1992. It is an action agenda for
the UN, other multilateral organizations, and individual governments around the world that can
be executed at local, national, and global levels. The "21" in Agenda 21 refers to the
21st Century. It has been affirmed and had a few modifications at subsequent UN conferences.

QUESTION 44. MTczODc4K1JhaHVsIFNyZWVkaGFyK3JhaHVsc3JlZWRoYXI4Nzg3QGdtYWlsLmNvbStRVUVTV


lPTiA0Mw==
Since the solid waste generation is rising in urban areas of India, many waste to energy plants
are being started. They serve the dual purpose of waste disposal as well as generation of
electricity. Which of the following statements are correct about waste to energy generation?

1. The cost of waste to energy is somewhat higher than other renewable sources.

2. The chimneys of incinerators include acid gases, nitrogen oxide, heavy metals, particulates,
and dioxin, which is a carcinogen.

3. The efficiency of waste to energy plants is very low.

Select the correct code from the following:

a) 1 and 2
b) 2 and 3
c) 1 and 3
d) All of the above
Correct Answer: D

Explanation

Solution (d)

While the Indian Governments own figures would suggest that the cost of waste to energy is
somewhat higher than other renewable sources, it is still an attractive option, as it serves a
dual role of waste disposal and energy production.

Smoke and ash emitted by the chimneys of incinerators include acid gases, nitrogen oxide,
heavy metals, particulates, and dioxin, which is a carcinogen. While incineration pollution
control technology is evolving to reduce these pollutants, it has been found that even with
controls in place, some remaining dioxin still enters the atmosphere.

IASbaba
Web: http://ilp.iasbaba.com/ Score:
Email: ilp@iasbaba.com 0.00 / 150
Page 33
Exam Title : 2019 - Test 15- Environ/S&T...
Email : rahulsreedhar8787@gmail.com
Contact :

QUESTION 45. MTczODc4K1JhaHVsIFNyZWVkaGFyK3JhaHVsc3JlZWRoYXI4Nzg3QGdtYWlsLmNvbStRVUVTV


ElPTiA0NA==
Consider the following statements:

1. Fly ash can be used to make fire resistant bricks.

2. Fly ash can be mixed with cement thus reducing the cost of construction.

3. Fly ash can cover the surface of plant leaves and act as a nutrient.

4. Fly ash bricks are light weight and offer high strength and durability.

Which of the above statements are the advantages of fly ash?

a) 1,2 and 3
b) 2,3 and 4
c) 1,2 and 4
d) All of the above
Correct Answer: C

Explanation

Solution (c)

Fly ash in atmosphere acts as a pollutant. It can cover the leaf surface by making a thin layer
which reduces photosynthesis and productivity of plants.

QUESTION 46. MTczODc4K1JhaHVsIFNyZWVkaGFyK3JhaHVsc3JlZWRoYXI4Nzg3QGdtYWlsLmNvbStRVUVTV


ElPTiA0NQ==
Mitochondria is an organelle found in most of the cells. It is known as the powerhouse of the
cell. Which of the following statements are correct related to mitochondria?

1. The biochemical processes of respiration and energy production occurs here.

2. Mitochondrial disease can pass to a progeny only through mothers in human beings.

3. Three parent baby is a technique used to prevent the transfer of mitochondrial defect to the
babies.

Select the correct code from the following:

a) 1 and 2
b) 2 and 3
c) 1 and 3
d) All of the above
Correct Answer: D
Explanation

Solution (d)

IASbaba
Web: http://ilp.iasbaba.com/ Score:
Email: ilp@iasbaba.com 0.00 / 150
Page 34
Exam Title : 2019 - Test 15- Environ/S&T...
Email : rahulsreedhar8787@gmail.com
Contact :

In a male germ cell (sperm), the tail part contains the mitochondria. As tale does not take part
in the process of fertilization, it is not passed to a progeny from fathers.

Mitochondrion is present in the cytoplasm of the mothers cells. In three parent baby, the
nucleus from mothers egg cell is extracted and planted in the cytoplasm of another womens egg
cell that does not have a mitochondrial defect. This new cell is fertilized by fathers sperm. Since
the genetic characteristics lie in the nucleus, the baby gets his genetic traits from two parents
only.

QUESTION 47. MTczODc4K1JhaHVsIFNyZWVkaGFyK3JhaHVsc3JlZWRoYXI4Nzg3QGdtYWlsLmNvbStRVUVTV


ElPTiA0Ng==
Heat from the earth can be used as an energy source in many ways, from large and complex
power stations to small and relatively simple pumping systems. This heat energy is known as
geothermal energy. Consider the following statements regarding geo thermal energy:

1. The predominant source of the Earth’s heat is the gradual decay of radioactive isotopes.

2. It has extensive global distribution and it is accessible in both developed as well as under
developed nations.

3. It has low emission of sulphur, CO2 and other greenhouse gases.

4. It is independent of external supply and demand effects and fluctuations in exchange rates.

Which of the above statements are correct?

a) 1,2 and 3
b) 2,3 and 4
c) 1,2 and 4
d) All of the above
Correct Answer: D

Explanation

Solution (d)

Heat from the earth can be used as an energy source in many ways, from large and complex
power stations to small and relatively simple pumping systems. This heat energy, known as
geothermal energy, can be found almost anywhereas far away as remote deep wells in
Indonesia and as close as the dirt in our backyards.

Many regions of the world are already tapping geothermal energy as an affordable and
sustainable solution to reducing dependence on fossil fuels, and the global warming and public
health risks that result from their use.

The general characteristics of geothermal energy that make it of significant importance for both
electricity production and direct use include:

• Extensive global distribution; it is accessible to both developed and developing countries.


• Environmentally friendly nature; it has low emission of sulphur, CO2 and other
greenhouse gases.

IASbaba
Web: http://ilp.iasbaba.com/ Score:
Email: ilp@iasbaba.com 0.00 / 150
Page 35
Exam Title : 2019 - Test 15- Environ/S&T...
Email : rahulsreedhar8787@gmail.com
Contact :

• Indigenous nature; it is independent of external supply and demand effects and


fluctuations in exchange rates.
• Independence of weather and season.
• Contribution to the development of diversified power sources.

QUESTION 48. MTczODc4K1JhaHVsIFNyZWVkaGFyK3JhaHVsc3JlZWRoYXI4Nzg3QGdtYWlsLmNvbStRVUVTV


ElPTiA0Nw==
Solar energy is radiant light and heat from the Sun that is harnessed using a range of ever-
evolving technologies. Solar power is the conversion of sunlight into electricity, either directly
using photovoltaics (PV), or indirectly using concentrated solar power (CSP). Consider the
following statements regarding Solar Power Technology:

1. Photovoltaic cells use the ultraviolet radiations to convert solar energy into electric current.

2. Semiconductors (usually silicon) are used to manufacture solar panels.

3. Concentrated Solar power use infrared radiations to heat the water and rotate turbines with
the generated steam.

4. Concentrated Solar Power systems generally use a huge convex lens to concentrate energy at
its focus.

Which of the above statements are correct?

a) 1 and 2
b) 3 and 4
c) 2 and 3
d) 1,2 and 3
Correct Answer: C

Explanation

Solution (c)

Photovoltaic cells use photons or light radiations to produce electric current.

Concentrated Solar Power systems usually use large set of plane mirrors or concave mirrors to
focus sunlight.

QUESTION 49. MTczODc4K1JhaHVsIFNyZWVkaGFyK3JhaHVsc3JlZWRoYXI4Nzg3QGdtYWlsLmNvbStRVUVTV


ElPTiA0OA==
Buildings contribute maximum to the greenhouse gas emissions in the world. To reduce the
carbon foot print of buildings, the concept of green buildings was given. Consider the following
regarding Green Buildings:

1. Green building refers to both a structure and the using of processes that are environmentally
responsible and resource-efficient throughout a building's life-cycle.

2. GRIHA , an acronym for Green Rating for Integrated Habitat Assessment , is the National
Green Building Rating System of India.

IASbaba
Web: http://ilp.iasbaba.com/ Score:
Email: ilp@iasbaba.com 0.00 / 150
Page 36
Exam Title : 2019 - Test 15- Environ/S&T...
Email : rahulsreedhar8787@gmail.com
Contact :

3. GRIHA has been developed by TERI.

Which of the above statements are correct?

a) 1 and 2
b) 2 and 3
c) 1 and 3
d) All of the above
Correct Answer: D

Explanation

Solution (d)

Green building (also known as green construction or sustainable building ) refers to both
a structure and the using of processes that are environmentally responsible and resource-
efficient throughout a building's life-cycle: from siting to design, construction, operation,
maintenance, renovation, and demolition. In other words, green building design involves finding
the balance between homebuilding and the sustainable environment. This requires close
cooperation of the design team, the architects, the engineers, and the client at all project
stages. The Green Building practice expands and complements the classical building design
concerns of economy, utility, durability, and comfort.

Leadership in Energy and Environmental Design (LEED) is a set of rating systems for the
design, construction, operation, and maintenance of green buildings which was Developed by
the U.S. Green Building Council.

GRIHA, an acronym for Green Rating for Integrated Habitat Assessment , is the National
Rating System of India. TERI took the responsibility of popularizing green building by
developing a tool for measuring and rating a building's environmental performance in the
context of India's varied climate and building practices.

QUESTION 50. MTczODc4K1JhaHVsIFNyZWVkaGFyK3JhaHVsc3JlZWRoYXI4Nzg3QGdtYWlsLmNvbStRVUVTV


ElPTiA0OQ==
Which of the following statements correctly explains an Urban Heat Island?

a) A city or a metropolitan area, which is significantly warmer than the surrounding rural
area, due to anthropogenic activities.
b) An island with sudden increase in urbanization activity, increasing its average temperature.
c) A volcanic island where people have settled due to population pressure on the mainland.
d) None of the above
Correct Answer: A

Explanation

Solution (a)

An urban heat island ( UHI ) is a city or metropolitan area that is significantly warmer than
its surrounding rural areas due to human activities. The temperature difference usually is
larger at night than during the day, and is most apparent when winds are weak. The main cause

IASbaba
Web: http://ilp.iasbaba.com/ Score:
Email: ilp@iasbaba.com 0.00 / 150
Page 37
Exam Title : 2019 - Test 15- Environ/S&T...
Email : rahulsreedhar8787@gmail.com
Contact :

of the urban heat island effect is from the modification of land surfaces. Waste heat generated
by energy usage is a secondary contributor. As a population center grows, it tends to expand its
area and increase its average temperature. The less-used term heat island refers to any area,
populated or not, which is consistently hotter than the surrounding area.

QUESTION 51. MTczODc4K1JhaHVsIFNyZWVkaGFyK3JhaHVsc3JlZWRoYXI4Nzg3QGdtYWlsLmNvbStRVUVTV


lPTiA1MA==
‘Exercise Dharma Guardian’ is bilateral military exercise between India and

a) Japan
b) France
c) United States of America
d) Russia
Correct Answer: A

Explanation

Solution (a)

Exercise Dharma Guardian

To promote Military cooperation, India and Japan are all set to hold the first ever joint military
exercise 'DHARMA GUARDIAN-2018' involving the Indian Army and Japan Ground Self Defence
Force 173878

QUESTION 52. MTczODc4K1JhaHVsIFNyZWVkaGFyK3JhaHVsc3JlZWRoYXI4Nzg3QGdtYWlsLmNvbStRVUVTV


lPTiA1MQ==
Consider the following statements with respect to ‘ROSHNI’

1. It is under the aegis of the Ministry of Women and Child Development

2. It is a centre of Women Collectives led Social Action financially supported by UNICEF India

Select the correct statements

a) 1 Only
b) 2 Only
c) Both 1 and 2
d) Neither 1 nor 2
Correct Answer: B

Explanation

Solution (b)

ROSHNI Centre of Women Collectives led Social Action

IASbaba
Web: http://ilp.iasbaba.com/ Score:
Email: ilp@iasbaba.com 0.00 / 150
Page 38
Exam Title : 2019 - Test 15- Environ/S&T...
Email : rahulsreedhar8787@gmail.com
Contact :

News: Deendayal Antyodaya Yojana National Rural Livelihood Mission (DAY-NRLM), Ministry of
Rural Development and Lady Irwin College, have signed a Memorandum of Understanding
(MoU) for establishing ROSHNI

About

It aims to work with women collectives for social action on Food, Health, Nutrition and WASH
(FHNW) interventions in line with NRLMs strategy.

It supports implementation of action plan under POSHAN Abhiyaan.

It is under the ministrys National Rural Livelihood Mission (NRLM).

It is technically and financially supported by UNICEF India.

QUESTION 53. MTczODc4K1JhaHVsIFNyZWVkaGFyK3JhaHVsc3JlZWRoYXI4Nzg3QGdtYWlsLmNvbStRVUVTV


ElPTiA1Mg==
Consider the following statements with respect to ‘Organophosphates’

1. They are widely used as pesticides and insecticides

2. It is used to control adult mosquitoes to combat an outbreak of mosquito-borne disease

Select the correct statements

a) 1 Only
b) 2 Only
c) Both 1 and 2
d) Neither 1 nor 2
Correct Answer: C

Explanation

Solution (c)

Organophosphate-based pesticides

Organophosphate-based pesticides, which are commonly used by farmers in India, are toxic to
the nervous system and heart, and can cause cognitive dysfunction.

When esters present in organophosphate-based pesticides enter the body they bind and inhibit
an enzyme (acetylcholinesterase or AChE) critical for nerve and muscle function.

This causes neurological disorders, suffocation, paralysis, and even death.

Majority of organophosphate-based pesticides are absorbed through the skin (the nasal/
inhalation route constitutes about 10-15%)

In addition to their use in agriculture, organophosphates are also used in public health
applicationslike the elimination of the mosquitoes that harbor the West Nile virusin some
countries.

IASbaba
Web: http://ilp.iasbaba.com/ Score:
Email: ilp@iasbaba.com 0.00 / 150
Page 39
Exam Title : 2019 - Test 15- Environ/S&T...
Email : rahulsreedhar8787@gmail.com
Contact :

Upon entering the bodythrough ingestion, inhalation, or contact with skinorganophosphates


inhibit cholinesterase, an enzyme in the human nervous system that breaks down acetylcholine,
a neurotransmitter that carries signals between nerves and muscles.

When cholinesterase is inactivated, acetylcholine builds up in the nerves, which become


overactive. Victims of organophosphate poisoning typically die because they can't breathe.

QUESTION 54. MTczODc4K1JhaHVsIFNyZWVkaGFyK3JhaHVsc3JlZWRoYXI4Nzg3QGdtYWlsLmNvbStRVUVTV


lPTiA1Mw==
Consider the following statements with respect to ‘Institute for Stem Cell Biology and
Regenerative Medicine ( inStem )’

1. It is an autonomous institute under the Department of Biotechnology

2. It is a bilateral collaboration between the India and Norway

Select the correct statements

a) 1 Only
b) 2 Only
c) Both 1 and 2
d) Neither 1 nor 2
Correct Answer: A

Explanation

Solution (a)

Institute for Stem Cell Biology and Regenerative Medicine (inStem)

It is an autonomous research institute in Bangalore, dedicated to the study of stem cell biology
and regenerative medicine research.

inStem is funded by Department of Biotechnology and well-supported, with access to facilities


at both National Centre for Biological Sciences (NCBS) and Centre for Cellular and Molecular
Platforms (C-CAMP).

Together these three institutions serve as part of the Bangalore Bio-Cluster. The institute is also
the umbrella organization for three initiatives: inStem itself, the Center for Stem Cell Research
(CSCR) located at CMC Vellore, and an Extramural Program in Stem Cell Research (EPiSTEM),
a funding initiative for support of stem cell research nationwide.

QUESTION 55. MTczODc4K1JhaHVsIFNyZWVkaGFyK3JhaHVsc3JlZWRoYXI4Nzg3QGdtYWlsLmNvbStRVUVTV


ElPTiA1NA==
Consider the following statements with respect to ‘Petroglyphs’

1. They are images created by removing part of a rock surface by incising, picking, or carving

2. In India Petroglyphs are found only in Southern India

IASbaba
Web: http://ilp.iasbaba.com/ Score:
Email: ilp@iasbaba.com 0.00 / 150
Page 40
Exam Title : 2019 - Test 15- Environ/S&T...
Email : rahulsreedhar8787@gmail.com
Contact :

Select the correct statements

a) 1 Only
b) 2 Only
c) Both 1 and 2
d) Neither 1 nor 2
Correct Answer: A

Explanation

Solution (a)

Petroglyphs are images created by removing part of a rock surface by incising, picking, carving,
or abrading, as a form of rock art. Petroglyphs are found worldwide, and are often associated
with prehistoric peoples.

In India, they are discovered in Karnataka, Madhya Pradesh, Maharashtra, Tripura, Ladakh etc.

Petroglyphs of Ratnagiri In News

1,000 rock carvings were recently discovered in and around the Ratnagiri and Rajapur districts
of Maharashtras Konkan coast

It is expected to provide new insights into the early history of the region.

Petroglyphs are images created by removing part of a rock surface by incising, picking, carving,
or abrading, as a form of rock art.

Findings

These sites contained an incredible range of images, from basic depictions of human and animal
forms to a 50-ft carving of an elephant, within which a series of smaller animal and aquatic
forms were drawn.

Abstract patterns and fertility symbols carved rudimentarily on the rock surface

Complex geometric reliefs cut deep into the rock.

The working theory around these petroglyphs is that they date back to about 10,000 BCE,
placing them in the Mesolithic Period, which comes between the Old Stone Age or Paleolithic
period, characterised by chipped stone tools, and the New Stone Age or Neolithic period,
associated with smaller, more polished tools.

QUESTION 56. MTczODc4K1JhaHVsIFNyZWVkaGFyK3JhaHVsc3JlZWRoYXI4Nzg3QGdtYWlsLmNvbStRVUVTV


ElPTiA1NQ==
‘Kimberly Process Certificate Scheme (KPCS)’ is concerned with

a) Immigration
b) Diamonds
c) Intellectual Property
d) Renewable Energy

IASbaba
Web: http://ilp.iasbaba.com/ Score:
Email: ilp@iasbaba.com 0.00 / 150
Page 41
Exam Title : 2019 - Test 15- Environ/S&T...
Email : rahulsreedhar8787@gmail.com
Contact :

Correct Answer: B

Explanation

Solution (b)

Kimberly Process Certification Scheme

The Kimberley Process is a joint Government, International Diamond Industry and Civil Society
initiative to stem the flow of Conflict Diamonds.

Conflict Diamonds means rough diamonds used by rebel movements or their allies to finance
conflict aimed at undermining legitimate governments.

It is also described in the United Nations Security Council (UNSC) resolutions.

The KPCS came to into effect from 1st January, 2003 and evolved into an effective mechanism
for stopping the trade in conflict diamonds.

India is the founding member of KPCS.

India is one of the founding members of the KPCS and was Chair of the KPCS for the year 2008.

India is appointed as the Kimberly Process (KP) Vice Chair for 2018 & Chair for 2019

QUESTION 57. MTczODc4K1JhaHVsIFNyZWVkaGFyK3JhaHVsc3JlZWRoYXI4Nzg3QGdtYWlsLmNvbStRVUVTV


ElPTiA1Ng==
Consider the following statements with respect to ‘ Similipal National Park’

1. It is a significant wildlife corridor in the Nilgiri Biosphere Reserve between the Western
Ghats and the Eastern Ghats

2. It is the largest among the four tiger reserves in Tamil Nadu

Select the correct statements

a) 1 Only
b) 2 Only
c) Both 1 and 2
d) Neither 1 nor 2
Correct Answer: D

Explanation

Solution (d)

Similipal National Park

It is a national park and a tiger reserve in the Mayurbhanj district in Odisha.

It is part of the Similipal-Kuldiha-Hadgarh Elephant Reserve popularly known as Mayurbhanj


Elephant Reserve, which includes three protected areas Similipal Tiger Reserve, Hadgarh
Wildlife Sanctuary and Kuldiha Wildlife Sanctuary.

IASbaba
Web: http://ilp.iasbaba.com/ Score:
Email: ilp@iasbaba.com 0.00 / 150
Page 42
Exam Title : 2019 - Test 15- Environ/S&T...
Email : rahulsreedhar8787@gmail.com
Contact :

Simlipal National Park derives its name from the abundance of semul (red silk cotton trees) that
bloom here.

QUESTION 58. MTczODc4K1JhaHVsIFNyZWVkaGFyK3JhaHVsc3JlZWRoYXI4Nzg3QGdtYWlsLmNvbStRVUVTV


ElPTiA1Nw==
Which of the following is/are correctly matched?

Tribes States

1. Mankidia Odisha

2. Agariya Gujarat

3. Chakma Andaman Islands

Select the correct code:

a) 1 and 2
b) 2 and 3
c) 1 and 3
d) 1 Only
Correct Answer: A

Explanation

Solution (a)

The Chakmas, also known as the Changma, Daingnet people, are an ethnic group scattered in
Arunachal Pradesh, Tripura, Assam, Mizoram, Meghalaya and West Bengal of India and in
Chittagong Hill Tracts of Bangladesh.

There is a group known as the Agariya in Gujarat that are salt makers in the desert. An
interesting ritual associated with this community is that the feet of agariya people are burnt
separately. Since they are standing continuously in salt fields, their feet get wounded and salt
get absorbed in the feet. So it will not burn easily in the funeral. It is not clear if these Agariya
have any relation to the others.

QUESTION 59. MTczODc4K1JhaHVsIFNyZWVkaGFyK3JhaHVsc3JlZWRoYXI4Nzg3QGdtYWlsLmNvbStRVUVTV


ElPTiA1OA==
Consider the following statements with respect to ‘Commission of Railway Safety’

1. It is an independent body under the Union Ministry of Civil Aviation, investigates serious
train accidents.

2. It was formed aftermath of ‘Bihar train derailment’ of 1981.

Select the correct statements

a) 1 Only

IASbaba
Web: http://ilp.iasbaba.com/ Score:
Email: ilp@iasbaba.com 0.00 / 150
Page 43
Exam Title : 2019 - Test 15- Environ/S&T...
Email : rahulsreedhar8787@gmail.com
Contact :

b) 2 Only
c) Both 1 and 2
d) Neither 1 nor 2
Correct Answer: A

Explanation

Solution (a)

Commission of Railway Safety

It is an independent body under the Union Ministry of Civil Aviation, investigates serious train
accidents.

It deals with matters pertaining to safety of rail travel and train operation and is charged with
certain statutory functions as laid down in the Railways Act (1989), which are of an inspectorial,
investigatory & advisory nature.

The Commission functions according to certain rules viz. Statutory investigation into accidents
rules framed under the Railways Act and executive instructions issued from time to time.

Its head office is in the North-East Railway Compound in Lucknow.

It was fomed in 1961

Source: https://www.thehindu.com/todays-paper/tp-national/no-nia-chargesheet-in-
derailment-case/article25282182.ece

QUESTION 60. MTczODc4K1JhaHVsIFNyZWVkaGFyK3JhaHVsc3JlZWRoYXI4Nzg3QGdtYWlsLmNvbStRVUVTV


ElPTiA1OQ==
‘Bedouin’ is associated with which of the following?

a) South East Asia


b) Eastern Europe
c) Yucatan Peninsula
d) None of the above
Correct Answer: D

Explanation

Solution (d)

The Bedouin or Bedu are a grouping of nomadic Arab people who have historically inhabited
the desert regions in North Africa, the Arabian Peninsula, Iraq and the Levant.

Unrecognized Bedouin villages in Israel are rural Bedouin communities in the Negev and the
Galilee which the Israeli government does not recognize as legal. Often they are referred to as
"unrecognized villages".

IASbaba
Web: http://ilp.iasbaba.com/ Score:
Email: ilp@iasbaba.com 0.00 / 150
Page 44
Exam Title : 2019 - Test 15- Environ/S&T...
Email : rahulsreedhar8787@gmail.com
Contact :

QUESTION 61. MTczODc4K1JhaHVsIFNyZWVkaGFyK3JhaHVsc3JlZWRoYXI4Nzg3QGdtYWlsLmNvbStRVUVTV


lPTiA2MA==
‘ Dzükou Valley’ is a valley located at the border of

a) Punjab and Pakistan


b) Nepal and China
c) Nagaland and Manipur
d) Arunachal Pradesh and China
Correct Answer: C

Explanation

Solution (c)

The Dzkou Valley is a valley located at the border of the states of Nagaland and Manipur. This
valley is well known for its natural environment, seasonal flowers and flora and fauna.

THINK!

Ziro Valley

QUESTION 62. MTczODc4K1JhaHVsIFNyZWVkaGFyK3JhaHVsc3JlZWRoYXI4Nzg3QGdtYWlsLmNvbStRVUVTV


lPTiA2MQ==
The term, ‘Representative Concentration Pathways’ is associated with

a) Green Rail Corridor


b) Eco-bridges for tigers
c) Goods and Services Tax
d) None of the above
Correct Answer: D

Explanation

Solution (d)

Representative Concentration Pathways (RCPs) are four greenhouse gas concentration (not
emissions) trajectories adopted by the IPCC for its fifth Assessment Report (AR5) in 2014. It
supersedes Special Report on Emissions Scenarios (SRES) projections published in 2000.

QUESTION 63. MTczODc4K1JhaHVsIFNyZWVkaGFyK3JhaHVsc3JlZWRoYXI4Nzg3QGdtYWlsLmNvbStRVUVTV


ElPTiA2Mg==
‘Cocoa butter equivalent or CBE’ is made from the extracts of which of the following trees?

1. Kokum

2. Mahua

3. Dhupa

IASbaba
Web: http://ilp.iasbaba.com/ Score:
Email: ilp@iasbaba.com 0.00 / 150
Page 45
Exam Title : 2019 - Test 15- Environ/S&T...
Email : rahulsreedhar8787@gmail.com
Contact :

Select the correct code:

a) 1 and 2
b) 2 and 3
c) 1 and 3
d) All of the above
Correct Answer: D

Explanation

Solution (d)

Cocoa butter equivalent or CBE

It is a kind of fat and has characteristics similar to that of the much more expensive cocoa
butter with which it can be mixed in any proportion without the cocoa butter losing its
properties or formulation.

A prime reason for the use of CBE is that it costs almost half when compared with the edible
vegetable fat extracted from cocoa beans.

It is made from the extracts of tree-borne butter seeds such as sal, mango, kokum, mahua,
dhupa and shea that is collected from the jungles of Jharkhand, Odisha, Chhattisgarh and
Madhya Pradesh

QUESTION 64. MTczODc4K1JhaHVsIFNyZWVkaGFyK3JhaHVsc3JlZWRoYXI4Nzg3QGdtYWlsLmNvbStRVUVTV


lPTiA2Mw==
Consider the following statements with respect to ‘ Baishui No. 1 Glacier’

1. It is a long ice shelf in the northwest part of the Weddell Sea

2. It extends along the east coast of the Antarctic Peninsula

Select the correct statements

a) 1 Only
b) 2 Only
c) Both 1 and 2
d) Neither 1 nor 2
Correct Answer: D

Explanation

Solution (d)

Baishui No. 1 Glacier

It is on the south eastern edge of the Third Pole a region in Central Asia with the worlds third
largest store of ice after Antarctica and Greenland thats roughly the size of Texas and New
Mexico combined.

IASbaba
Web: http://ilp.iasbaba.com/ Score:
Email: ilp@iasbaba.com 0.00 / 150
Page 46
Exam Title : 2019 - Test 15- Environ/S&T...
Email : rahulsreedhar8787@gmail.com
Contact :

Third Pole glaciers are vital to billions of people from Vietnam to Afghanistan. Asias 10 largest
rivers including the Yangtze, Yellow, Mekong, and Ganges are fed by seasonal melting.

The glacier has lost 60% of its mass and shrunk 250 m since 1982

QUESTION 65. MTczODc4K1JhaHVsIFNyZWVkaGFyK3JhaHVsc3JlZWRoYXI4Nzg3QGdtYWlsLmNvbStRVUVTV


ElPTiA2NA==
‘ Novator 9M729’ was in news recently. What is it?

a) Frigate develop by India


b) Largest Unmanned Aerial Vehicle developed by China
c) Military drone developed jointly developed by India and Israel
d) None of the above
Correct Answer: D

Explanation

Solution (d)

Novator 9M729

The land-based cruise missile is thought to have a range that falls between 500km and
5,500km, and is therefore illegal under the terms of the INF treaty.

It carries the NATO designation of SSC-8.

QUESTION 66. MTczODc4K1JhaHVsIFNyZWVkaGFyK3JhaHVsc3JlZWRoYXI4Nzg3QGdtYWlsLmNvbStRVUVTV


ElPTiA2NQ==
Consider the following statements about ‘INF Treaty’

1. It required the United States and the Soviet Union to eliminate and permanently forswear all
of their nuclear and conventional ground-launched ballistic and cruise missiles with ranges of
500 to 5,500 kilometres.

2. The INF ban originally applied only to U.S. and Soviet forces, but the treaty's membership
expanded in 1991 to include successor states of the former Soviet Union.

Select the correct statements

a) 1 Only
b) 2 Only
c) Both 1 and 2
d) Neither 1 nor 2
Correct Answer: C
Explanation

IASbaba
Web: http://ilp.iasbaba.com/ Score:
Email: ilp@iasbaba.com 0.00 / 150
Page 47
Exam Title : 2019 - Test 15- Environ/S&T...
Email : rahulsreedhar8787@gmail.com
Contact :

Solution (c)

INF Treaty

News: US to withdraw from the treaty, due to Russias alleged development and deployment of
the Novator 9M729 missile

About

The Intermediate-Range Nuclear Forces Treaty (INF Treaty, formally Treaty Between the
United States of America and the Union of Soviet Socialist Republics on the Elimination of Their
Intermediate-Range and Shorter-Range Missiles) is a 1987 arms control agreement between
the United States and the Soviet Union (and later its successor state, in particular the Russian
Federation).

The treaty was ratified by the United States Senate on 27 May 1988 and came into force on 1
June 1988.

It is a crucial Cold War-era treaty banning the development, testing and possession of short and
medium range ground-launched nuclear missiles with a range of 500-5,000 km.

The treaty was central to ending the arms race between the two superpowers, and protected
Americas NATO allies in Europe from Soviet missile attacks.

The INF ban originally applied only to U.S. and Soviet forces, but the treaty's membership
expanded in 1991 to include successor states of the former Soviet Union. Today, Belarus,
Kazakhstan, and Ukraine join Russia and the United States in the treaty's implementation.
Turkmenistan and Uzbekistan possessed INF facilities (SS-23 operating bases) but forgo treaty
meetings with the consent of the other states-parties.

Although active states-parties to the treaty total just five countries, several European countries
have destroyed INF-banned missiles since the end of the Cold War.

Germany, Hungary, Poland, and the Czech Republic destroyed their intermediate-range missiles
in the 1990s, and Slovakia dismantled all of its remaining intermediate-range missiles in
October 2000 after extensive U.S. prodding.

On May 31, 2002, the last possessor of intermediate-range missiles in eastern Europe, Bulgaria,
signed an agreement with the United States to destroy all of its INF Treaty-relevant missiles.
Bulgaria completed the destruction five months later with U.S. funding.

QUESTION 67. MTczODc4K1JhaHVsIFNyZWVkaGFyK3JhaHVsc3JlZWRoYXI4Nzg3QGdtYWlsLmNvbStRVUVTV


ElPTiA2Ng==
Consider the following statements about ‘Green Climate Fund (GCF)’

1. It is a fund established within the framework of the UNFCCC as an operating entity of the
Financial Mechanism to assist developing countries in adaptation and mitigation practices to
counter climate change.

2. It was established during the 2012 United Nations Climate Change Conference in Doha

Select the correct statements

IASbaba
Web: http://ilp.iasbaba.com/ Score:
Email: ilp@iasbaba.com 0.00 / 150
Page 48
Exam Title : 2019 - Test 15- Environ/S&T...
Email : rahulsreedhar8787@gmail.com
Contact :

a) 1 Only
b) 2 Only
c) Both 1 and 2
d) Neither 1 nor 2
Correct Answer: A

Explanation

Solution (a)

Green Climate Fund

News: Green Climate Fund invests USD 1 billion for developing country climate action,
launches first replenishment

About

It is a fund established within the framework of the UNFCCC as an operating entity of the
Financial Mechanism to assist developing countries in adaptation and mitigation practices to
counter climate change.

The GCF is based in the Incheon, South Korea. It is governed by a Board of 24 members and
supported by a Secretariat.

The Fund is a unique global platform to respond to climate change by investing in low-emission
and climate-resilient development.

GCF was established to limit or reduce greenhouse gas (GHG) emissions in developing
countries, and to help vulnerable societies adapt to the unavoidable impacts of climate change.

Given the urgency and seriousness of this challenge, the Fund is mandated to make an
ambitious contribution to the united global response to climate change.

The Copenhagen Accord, established during the 2009 United Nations Climate Change
Conference (COP-15) in Copenhagen mentioned the "Copenhagen Green Climate Fund".

The fund was formally established during the 2010 United Nations Climate Change Conference
in Cancun as a fund within the UNFCCC framework.

QUESTION 68. MTczODc4K1JhaHVsIFNyZWVkaGFyK3JhaHVsc3JlZWRoYXI4Nzg3QGdtYWlsLmNvbStRVUVTV


ElPTiA2Nw==
Consider the following statements about ‘ Neelakurinji ’

1. It blossoms only once in 12 years

2. It is endemic to Western Ghats

Select the correct statements

a) 1 Only
b) 2 Only
c) Both 1 and 2

IASbaba
Web: http://ilp.iasbaba.com/ Score:
Email: ilp@iasbaba.com 0.00 / 150
Page 49
Exam Title : 2019 - Test 15- Environ/S&T...
Email : rahulsreedhar8787@gmail.com
Contact :

d) Neither 1 nor 2
Correct Answer: C

Explanation

Solution (c)

Kurinji or Neelakurinji (Strobilanthes kunthianus) is a shrub that is found in the shola forests of
the Western Ghats in South India. Nilgiri Hills, which literally means the blue mountains, got
their name from the purplish blue flowers of Neelakurinji that blossoms only once in 12 years.

QUESTION 69. MTczODc4K1JhaHVsIFNyZWVkaGFyK3JhaHVsc3JlZWRoYXI4Nzg3QGdtYWlsLmNvbStRVUVTV


ElPTiA2OA==
‘ Migingo Island’ is disputed between which of the following coutries ?

a) China and Indonesia


b) Vietnam and Cambodia
c) Japan and Russia
d) Kenya and Uganda
Correct Answer: D

Explanation

Solution (d)

Migingo Island

It is claimed both by Kenya and Uganda

It is a 2,000-square-metre island, in Lake Victoria

QUESTION 70. MTczODc4K1JhaHVsIFNyZWVkaGFyK3JhaHVsc3JlZWRoYXI4Nzg3QGdtYWlsLmNvbStRVUVTV


ElPTiA2OQ==
Consider the following statements with respect to ‘UNCITRAL Model Law on Cross-Border
Insolvency’

1. It focuses on eliminating preferences for local creditors over international ones

2. It ensures that states give assistance to insolvency officials from other countries in relation to
main proceedings and non-main proceedings

3. India adopted the Model law in 1991 during the initiation of Liberalization, Privatization and
Globalization (LPG) Reforms

Select the correct statements

a) 1 and 2
b) 2 and 3

IASbaba
Web: http://ilp.iasbaba.com/ Score:
Email: ilp@iasbaba.com 0.00 / 150
Page 50
Exam Title : 2019 - Test 15- Environ/S&T...
Email : rahulsreedhar8787@gmail.com
Contact :

c) 1 and 3
d) All of the above
Correct Answer: A

Explanation

Solution (a)

UNCITRAL Model Law on Cross-Border Insolvency

News:

Insolvency Law Committee (ILC) constituted by the Ministry of Corporate Affairs to recommend
amendments to Insolvency and Bankruptcy Code of India, 2016, has submitted its 2nd Report to
the Government, which deals with cross border insolvency.

It has recommended the adoption of the United Nations Commission on International Trade
Law (UNCITRAL) Model Law of Cross Border Insolvency, 1997

About

It is a model law issued by the secretariat of UNCITRAL on 30 May 1997 to assist states in
relation to the regulation of corporate insolvency and financial distress involving companies
which have assets or creditors in more than one state.

It provides for a comprehensive framework to deal with cross-border insolvency issues

It has been adopted in 44 countries and, therefore, forms part of international best practices in
dealing with cross border insolvency issues

The advantages of the model law are the precedence given to domestic proceedings and
protection of public interest

The necessity of having a cross-border insolvency framework under the Insolvency and
Bankruptcy Code arises from the fact that many Indian companies have a global footprint and
many foreign companies have a presence in multiple countries, including India

The basic concept of the Model Law is to establish what the "main proceedings" are in relation
to any international insolvency.

All other proceedings are referred to as the "non-main proceedings". The main proceedings are
commenced where the debtor has its centre of main interest, or "COMI".

Non-main proceedings may be commenced in any place where the debtor has a commercial
establishment.

The Model Law does not require reciprocity between states, but focuses upon

ensuring that states give assistance to insolvency officials from other countries in relation to
main proceedings and non-main proceedings

eliminating preferences for local creditors over international ones.

Source: https://www.thehindu.com/business/panel-for-adopting-un-model-on-cross-
border-insolvency/article25290047.ece

IASbaba
Web: http://ilp.iasbaba.com/ Score:
Email: ilp@iasbaba.com 0.00 / 150
Page 51
Exam Title : 2019 - Test 15- Environ/S&T...
Email : rahulsreedhar8787@gmail.com
Contact :

QUESTION 71. MTczODc4K1JhaHVsIFNyZWVkaGFyK3JhaHVsc3JlZWRoYXI4Nzg3QGdtYWlsLmNvbStRVUVTV


lPTiA3MA==
The ‘Pearl River Delta’ includes which of the following regions?

1. Guangdong

2. Hog Kong

3. Macau

Select the correct statements

a) 1 and 2
b) 2 and 3
c) 1 and 3
d) All of the above
Correct Answer: D

Explanation

Solution (d)

Worlds longest sea bridge

It spans 55km (34 miles) and connects Hong Kong to Macau and the mainland Chinese city of
Zhuhai.

It is part of China's plan to create a Greater Bay Area, including Hong Kong, Macau and nine
other cities in southern China.

The bridge has been built to withstand super-typhoons, a magnitude-8 earthquake, as well as
hits by super-sized cargo ships.

About 30km of its total length crosses the sea of the Pearl River delta.

The Pearl River Delta Metropolitan Region, also known as Greater Bay, is the low-lying
area surrounding the Pearl River estuary, where the Pearl River flows into the South
China Sea. It is one of the most densely urbanized regions in the world.
Regions

Guangdong

Hong Kong

Macau

Major Settlements

Hong Kong

Macau

IASbaba
Web: http://ilp.iasbaba.com/ Score:
Email: ilp@iasbaba.com 0.00 / 150
Page 52
Exam Title : 2019 - Test 15- Environ/S&T...
Email : rahulsreedhar8787@gmail.com
Contact :

Guangzhou

Shenzhen

Dongguan

Foshan

Zhongshan

Jiangmen

Zhuhai

Huizhou

Zhaoqing

QUESTION 72. MTczODc4K1JhaHVsIFNyZWVkaGFyK3JhaHVsc3JlZWRoYXI4Nzg3QGdtYWlsLmNvbStRVUVTV


lPTiA3MQ==
Consider the following statements with respect to ‘International Conference on Status and
Protection of Coral Reefs (STAPCOR)

1. The STAPCOR 2018 was held at Bangaram Island in Lakshadweep

2. Ministry of Environment, Forest and Climate Change ( MoEFCC ) and International Union for
Conservation of Nature (IUCN) are part of the conference

Select the correct statements

a) 1 Only
b) 2 Only
c) Both 1 and 2
d) Neither 1 nor 2
Correct Answer: C

Explanation

Solution (c)

STAPCOR

The International Conference on Status and Protection of Coral Reefs (STAPCOR 2018) with the
theme Reef for Life was inaugurated

The Department of Environment and Forest, Union Territory of Lakshadweep Administration


organised this mega conference with the technical support of Zoological Survey of India and in
association with Ministry of Environment, Forest and Climate Change, IUCN, ENVIS in
consonance with declaration of the year 2018 as 3rd decadal International year of Reefs.

1st Edition 1998 (To have a international conference in every 10 years to review the status and
progress of coral reefs all over the world.)

IASbaba
Web: http://ilp.iasbaba.com/ Score:
Email: ilp@iasbaba.com 0.00 / 150
Page 53
Exam Title : 2019 - Test 15- Environ/S&T...
Email : rahulsreedhar8787@gmail.com
Contact :

QUESTION 73. MTczODc4K1JhaHVsIFNyZWVkaGFyK3JhaHVsc3JlZWRoYXI4Nzg3QGdtYWlsLmNvbStRVUVTV


ElPTiA3Mg==
Which of the following statements about ‘Glyphosate’ is/are correct?

a) It is used to regulate plant growth and


b) It is used to ripen fruit
c) Both (a) and (b)
d) Neither (a) nor (b)
Correct Answer: C

Explanation

Solution (c)

Glyphosate

News: Punjab government has banned the sale of glyphosate


About

It is an herbicide.

It is applied to the leaves of plants to kill both broadleaf plants and grasses.

The sodium salt form of glyphosate is used to regulate plant growth and ripen fruit.

It is a herbicide which is extensively used in Punjab to control a wide variety of weeds in almost
all the crops.

It has been observed to be a Group 2A cancer-causing material

It is also known for causing other health problems and has the potential to damage human DNA

DO YOU KNOW?

Central Insecticide Board and Registration Committee has recommended the use of the
herbicide only for tea gardens and non-cropped areas

QUESTION 74. MTczODc4K1JhaHVsIFNyZWVkaGFyK3JhaHVsc3JlZWRoYXI4Nzg3QGdtYWlsLmNvbStRVUVTV


lPTiA3Mw==
Consider the following statements with respect to ‘Fisheries and Aquaculture Infrastructure
Development Fund (FIDF)’

1. It entails a fund comprising completely of the Centre’s budgetary support of 1000 Crores

2. It would provide concessional finance to State Governments for taking up of the identified
investment activities of fisheries development.

Select the correct statements

IASbaba
Web: http://ilp.iasbaba.com/ Score:
Email: ilp@iasbaba.com 0.00 / 150
Page 54
Exam Title : 2019 - Test 15- Environ/S&T...
Email : rahulsreedhar8787@gmail.com
Contact :

a) 1 Only
b) 2 Only
c) Both 1 and 2
d) Neither 1 nor 2
Correct Answer: B

Explanation

Solution (b)

Fisheries and Aquaculture Infrastructure Development Fund (FIDF)

It entails an estimated fund size of Rs.7,522 crore, comprising Rs.5,266.40 crore to be raised by
the Nodal Loaning Entities (NLEs), Rs. 1,316.6 crore beneficiaries contribution and Rs.939.48
crore budgetary support from the Government of India.

National Bank for Agriculture and Rural Development (NABARD), National Cooperatives
Development Corporation (NCDC) and all scheduled Banks shall be the nodal Loaning Entities.

FIDF would provide concessional finance to State Governments / UTs and State entities,
cooperatives, individuals and entrepreneurs etc., for taking up of the identified investment
activities of fisheries development.

Under FIDF, loan lending will be over a period of five years from 2018-19 to 2022-23 and
maximum repayment will be over a period of 12 years inclusive of moratorium of two years on
repayment of principal.

Benefits:

Creation of fisheries infrastructure facilities both in marine and Inland fisheries sectors.

To augment fish production to achieve its target of 15 million tonne by 2020 set under the Blue
Revolution; and to achieve a sustainable growth of 8% -9% thereafter to reach the fish
production to the level of about 20 MMT by 2022-23.

Employment opportunities to over 9.40 lakh fishers/fishermen/fisherfolk and other


entrepreneurs in fishing and allied activities.

To attract private investment in creation and management of fisheries infrastructure facilities.

Adoption of new technologies.

QUESTION 75. MTczODc4K1JhaHVsIFNyZWVkaGFyK3JhaHVsc3JlZWRoYXI4Nzg3QGdtYWlsLmNvbStRVUVTV


ElPTiA3NA==
What is ‘ Hiyang Tannaba ’?

a) A traditional boat race of Manipur


b) Martial Art of Arunachal Pradesh
c) Colourful Motifs done with hands and paint made of rice and flour on auspicious occasions
in Mizoram.
d) An instrument played by Buddhist Monks

IASbaba
Web: http://ilp.iasbaba.com/ Score:
Email: ilp@iasbaba.com 0.00 / 150
Page 55
Exam Title : 2019 - Test 15- Environ/S&T...
Email : rahulsreedhar8787@gmail.com
Contact :

Correct Answer: A

Explanation

Solution (a)

Hiyang Tannaba is a traditional boat race of Manipur dating back to a few centuries and was
originally celebrated during the Manipuri Lunar month of Hiyangei Tha or October-November.
The race begins with a ritualistic ceremony to Lord Sanamahi or the ruling deity of every
Manipuri household, praying for safety and protection.

IASbaba
Web: http://ilp.iasbaba.com/ Score:
Email: ilp@iasbaba.com 0.00 / 150
Page 56

You might also like